You are on page 1of 96

Standardized GRADE 4

Test Tutor MATH

Michael Priestley
Standardized Test Tutor: Math, Grade 4 © Michael Priestley, Scholastic Teaching Resources
Scholastic Inc. grants teachers permission to photocopy the designated reproducible pages from this
book for classroom use. No other part of this publication may be reproduced in whole or in part,
or stored in a retrieval system, or transmitted in any form or by any means, electronic, mechanical,
photocopying, recording, or otherwise, without written permission of the publisher. For information
regarding permission, write to Scholastic Inc., 557 Broadway, New York, NY 10012.

Editor: Mela Ottaiano


Cover design: Brian LaRossa
Interior design: Creative Pages, Inc.
Interior illustrations: Creative Pages, Inc.

ISBN-13: 978-0-545-09606-5
ISBN-10: 0-545-09606-5
Copyright © 2009 by Michael Priestley
All rights reserved. Published by Scholastic Inc.
Printed in the U.S.A.

1 2 3 4 5 6 7 8 9 10 40 15 14 13 12 11 10 09

Standardized Test Tutor: Math, Grade 4 © Michael Priestley, Scholastic Teaching Resources
Contents
Welcome to Test Tutor . . . . . . . . . . . . . . . . . . . . . . . . . . . . . . . 4
Test 1 . . . . . . . . . . . . . . . . . . . . . . . . . . . . . . . . . . . . . . . . . . . . . . . . . . . . . . 8
Test 2 . . . . . . . . . . . . . . . . . . . . . . . . . . . . . . . . . . . . . . . . . . . . . . . . . . . . . 27
Test 3 . . . . . . . . . . . . . . . . . . . . . . . . . . . . . . . . . . . . . . . . . . . . . . . . . . . . . 45
Answer Sheet . . . . . . . . . . . . . . . . . . . . . . . . . . . . . . . . . . . . . . . . . . . . . 65

Answer Keys
Test 1 . . . . . . . . . . . . . . . . . . . . . . . . . . . . . . . . . . . . . . . . . . . . . . . . . . . . . 67
Test 2 . . . . . . . . . . . . . . . . . . . . . . . . . . . . . . . . . . . . . . . . . . . . . . . . . . . . . 76
Test 3 . . . . . . . . . . . . . . . . . . . . . . . . . . . . . . . . . . . . . . . . . . . . . . . . . . . . . 85

Scoring Charts
Student Scoring Chart . . . . . . . . . . . . . . . . . . . . . . . . . . . . . . . . . . . . . . 94
Classroom Scoring Chart . . . . . . . . . . . . . . . . . . . . . . . . . . . . . . . . . . . 95

Standardized Test Tutor: Math, Grade 4 © Michael Priestley, Scholastic Teaching Resources
Welcome to Test Tutor!
Students in schools today take a lot of tests, especially in reading and math. Some
students naturally perform well on tests, and some do not. But just about everyone
can get better at taking tests by learning more about what’s on the test and how to
answer the questions. How many students do you know who could benefit from
working with a tutor? How many would love to have someone sit beside them and
help them work their way through the tests they have to take?

That’s where Test Tutor comes in. The main purpose of Test Tutor is to help students
learn what they need to know in order to do better on tests. Along the way, Test
Tutor will help students feel more confident as they come to understand the content
and learn some of the secrets of success for multiple-choice tests.

The Test Tutor series includes books for reading and math in a range of grade levels.
Each Test Tutor book in mathematics has three full-length practice tests designed
specifically to resemble the state tests that students take each year. The math skills
measured on these practice tests have been selected from an analysis of the
skills tested in ten major states, and the questions have been written to match
the multiple-choice format used in most states.

The most important feature of this book is the friendly Test Tutor. He will
help students work through the tests and achieve the kind of success they
are looking for. This program is designed so students may work through
the tests independently by reading the Test Tutor’s helpful hints. Or you
may work with the student as a tutor yourself, helping him or her understand
each problem and test-taking strategy along the way. You can do this most
effectively by following Test Tutor’s guidelines included in the pages of
this book.

Three Different Tests


There are three practice tests in this book: Test 1, Test 2, and Test 3. Each test has
45 multiple-choice items with four answer choices (A, B, C, D). All three tests
measure the same skills in almost the same order, but they provide different levels
of tutoring help.

Test 1 provides step-by-step guidance to help students work through each problem,
as in the sample on the next page. The tips in Test 1 are detailed and thorough, and
they are written specifically for each math item to help students figure out how to
solve the problem.

4 Standardized Test Tutor: Math, Grade 4 © Michael Priestley, Scholastic Teaching Resources
Sample 1

S age is 5 feet 2 inches tall. Her brother Dylan is 4 feet


6 inches tall. How much taller is Sage than Dylan? To find how much taller
Sage is, you’ve got to
A 4 inches find the difference in their
heights. Subtract Dylan’s
B 6 inches
height from Sage’s height to
C 8 inches find how much taller she is.
D 1 foot 4 inches

Test 2 provides a test-taking tip for each item, as in the sample


below, but the tips are less detailed than in Test 1. They help guide
the student toward the solution to each problem without giving
away too much. Students must take a little more initiative.

Sample 2

I n 2006 the population of Aztec, New Mexico, was 7,056. In


1990 the population was 5,480. How much did the population Look for key words to
of Aztec grow from 1990 to 2006? help you understand the
question. In this question,
A 16 the key words are How
much and grow.
B 1,576
C 1,676
D 12,536

Test 3 does not provide test-taking tips. It assesses the progress


students have made. After working through Tests 1 and 2 with the
help of the Test Tutor, students should be more than ready to score
well on Test 3 without too much assistance. Success on this test will
help students feel confident and ready for taking real tests.

Standardized Test Tutor: Math, Grade 4 © Michael Priestley, Scholastic Teaching Resources
5
Other Helpful Features
In addition to the tests, this book provides some other helpful
features. First, on page 65, you will find an answer sheet. When
students take the tests, they may mark their answers by filling in
bubbles on the test pages. Or, they may mark their answers on
a copy of the answer sheet instead, as they will be required to
do in most standardized tests. You may want to have students
mark their answers on the test pages for Test 1 and then use an
answer sheet for Tests 2 and 3 to help the student get used to
filling in bubbles.

Second, beginning on page 67, you will find a detailed answer


key for each test. The answer key lists the correct response and explains
how to solve the problem. It also identifies the skill tested by each question,
as in the sample below.

Answer Key for Sample 2


Correct response: B
(Add, subtract, multiply, and divide whole numbers)
To find the difference in population, subtract the population in 1990
(5,480) from the population in 2006 (7,056): 7,056  5,480  1,576.

Incorrect choices:
A is the result of subtracting the years (2006  1990) instead of the
populations.
C is the result of a computation error in subtracting 7,056  5,480 as the
student “borrows” a hundred to subtract in the tens column but forgets
to reflect this in the hundreds column.
D is the result of adding 7,056  5,480 instead of subtracting.

As the sample answer indicates, this question measures the student’s ability
to add, subtract, multiply, and divide whole numbers. This information can
help you determine which skills the student has mastered and which ones still
cause difficulty.

Finally, the answer key explains why each incorrect answer choice, or “distractor,”
is incorrect. This explanation can help reveal what error students might have made.
For example, one distractor in an addition problem might be the result of subtracting
two numbers instead of adding them together. Knowing this could help the student
understand that he or she used the wrong operation.

6 Standardized Test Tutor: Math, Grade 4 © Michael Priestley, Scholastic Teaching Resources
At the back of this book, you will find two scoring charts. The Student Scoring
Chart can help you keep track of each student’s scores on all three tests and in
different subtests, such as “Number and Number Sense” or “Measurement and
Geometry.” The Classroom Scoring Chart can be used to record the scores for
all students on all three tests, illustrating the progress they have made from
Test 1 to Test 3. Ideally, students should score higher on each test as they go
through them. However, keep in mind that students get a lot of tutoring help
on Test 1, some help on Test 2, and no help on
Test 3. So, if a student’s scores on all
three tests are all fairly similar,
that could still be a very positive
sign that the student is better able
to solve problems independently
and will achieve even greater
success on future tests.

Standardized Test Tutor: Math, Grade 4 © Michael Priestley, Scholastic Teaching Resources
7
Test 1 Name Date

Directions: Read each question. Look at the Test Tutor’s tip for
answering the question. Then find the answer. You may do your
work on this page or on scrap paper. Mark your answer by filling Test
in the bubble. Tutor
says:

1. In 2006 the population of New York City was about eight
million two hundred fifty thousand. What is this number
in standard form? To write a number in
standard form, write the
A 8,250 number in the largest place
B 825,000 first. Then fill in the rest of
the numbers one part at a
C 8,000,250
time. For example, begin
D 8,250,000 with “eight million” as
8,000,000.

2. Four friends share a pizza. The table below shows what fraction
of the pizza each person ate.
Since all of these fractions
have 1 in the numerator
Amount of Pizza Eaten (top of fraction), compare
the amounts in the
Jerome ​ 1  ​ 
___
12 denominators (bottom of
Sheena 1 ​
​ __ fraction). Remember: The
3
denominator shows how
1 ​
​ __
Charlotte 4
many equal pieces a whole
1 ​
is divided into. So the
Bruce ​ __ smaller the denominator,
6
the bigger the piece.
Who ate the most pizza?
A Jerome
B Sheena
C Charlotte
D Bruce

8
Standardized Test Tutor: Math, Grade 4 © Michael Priestley, Scholastic Teaching Resources
Test 1 Name Test
Tutor
says:

3. Where is point M located on the number line?


M First, figure out how to read
0 1 the number line correctly.
Count the number of hash
A between 0.2 and 0.4 marks between 0 and 1.
Then you’ll know what each
B between 0.3 and 0.5
mark represents.
C between 0.1 and 0.2
D between 1 and 2

4. Ms. Ritchie buys 6 __​ 25 ​pounds of rice. What is another way to


write this number? To change a mixed number
to an improper fraction,
12  ​pounds
A ​ __
5
remember to multiply the
17
__
denominator by the whole
B ​  5  ​pounds number and then add that
30  ​pounds
C ​ __ product to the numerator.
5
32  ​pounds
D ​ __
5

5. In March, 628,451 letters were mailed from a post office. To find the place value of
Which digit in this number is in the ten thousands place? one digit in a number, write
out the number in expanded
A 2
form. This will tell you the
B 5 place value of each digit.
C 6 For example, 641 would be
600 + 40 + 1; the 4 in this
D 8 number is in the tens place.

6. There are 81 students in the fourth grade at Franklin School.


The principal wants to divide them into equal groups with no The number of groups
students left over. Into how many groups should the students must be a factor of 81. That
means that you can divide
be divided?
81 by that number, without
A 2 a remainder. Try dividing 81
B 3 by each answer choice.

C 7
D 11

9
Standardized Test Tutor: Math, Grade 4 © Michael Priestley, Scholastic Teaching Resources
Test 1 Name Test
Tutor
says:

7. On Monday, a grocery store had 8,120 cans of soup. On


Saturday, 6,529 of the cans were left. How many cans of To find the number of cans
soup were sold during the week? that were sold, subtract the
number of cans left over
A 1,591
on Saturday from the total
B 1,691 number of cans on Monday.
C 2,501
D 14,649

8. Sandy bought 2 __​ 34 ​pounds of green apples and 3 __​ 12 ​pounds


of red apples. How many pounds of apples did she buy First, find a common
denominator for the
all together?
fractions _​ 34 ​and _​ 12 ​. Then add
A ​ 14 ​pounds
5 __ the two mixed numbers to
find the total number of
B ​ 23 ​pounds
5 __
pounds of apples.
C 6 pounds

D ​ 14 ​pounds
6 __

9. Brody has a set of blocks. Each block is 6.4 centimeters tall. If he


stacks 8 blocks on top of one another, how tall will the stack of
blocks be? To find the height of the
stack, take the height of
A 48.2 centimeters each block and multiply it
B 482 centimeters by the number of blocks.

C 51.2 centimeters
D 512 centimeters

10
Standardized Test Tutor: Math, Grade 4 © Michael Priestley, Scholastic Teaching Resources
Test 1 Name Test
Tutor
says:

10. When Bonnie woke up at 7:00 a.m., the temperature was 1C.
When she got home from school at 3:00 p.m., the temperature
was 4C.

20 C
15
10
5
0
5
10

How many degrees did the temperature rise between 7:00 a.m.
and 3:00 p.m.? Subtract the morning
temperature from the
A 3C afternoon temperature.
B 4C Then you will know the
difference in degrees.
C 5C
Remember, 1C is
D 6C below 0.

11. It takes Latoya 28 minutes to make a holiday card. At this rate,
about how long will it take her to make 32 cards? “About how long” means
you can use estimation. To
A 10 hours estimate the answer, first
B 15 hours round each number to the
nearest ten. This will help
C 30 hours
you think in terms of hours
D 60 hours and half-hours.

12. Felicity has 4 boxes of cookies. Each box has 10 cookies.


To find the number of cookies she has in total (c), she writes Remember, multiplication is
the number sentence 4  10  c. Which number sentence repeated addition.
is equivalent to the one Felicity wrote?
A 4  10  c
B 4444c
C 10  10  10  10  c
D (4  10)  (4  10)  c

11
Standardized Test Tutor: Math, Grade 4 © Michael Priestley, Scholastic Teaching Resources
Test 1 Name Test
Tutor
says:

13. A pet store has 6 fish tanks with 30 fish in each tank. If __​ 13 ​of
the fish are goldfish, how many goldfish does the pet store Find the total number
have in all? of fish by multiplying first.
Then find one-third of that
A 10 number by dividing the
B 12 number by 3.

C 16
D 60

14. A bakery sells fruit bars for $1.25 each and muffins Write two number sentences
for $1.40 each. How much would 10 fruit bars and to solve this problem. You
12 muffins cost? will multiply the cost of
A $24.65 each item by the number
of items. Then find the sum.
B $29.00
C $29.30
D 58.30

15. Find the value of x in the number sentence below.


Think about what
15  x  5 number can you subtract
from 15 and have 5 left
A 3 over. If necessary, replace
B 5 the x with each answer
choice to see if the number
C 10 sentence makes sense.
D 20

12
Standardized Test Tutor: Math, Grade 4 © Michael Priestley, Scholastic Teaching Resources
Test 1 Name Test
Tutor
says:

16. Which digital clock tells the same time as the clock
shown below?

11 12 1
10 2
9 3 Look at each answer choice.
8 4 Which matches the time on
7 6 5 the clock face?

A C

B D

17. Ray’s house is 2,564 meters from Charlene’s house. How many
kilometers apart are their houses? Keep in mind that
one kilometer equals
A 0.2564 kilometers 1,000 meters.
B 2.564 kilometers
C 25.64 kilometers
D 256.4 kilometers

18. Georgia wants to find the weight of her 3-year-old dog. What
is the best unit of measurement for her to use? Remember that some
dogs can weigh as much
A tons
as children. What unit of
B ounces measurement do you use
to weigh yourself?
C grams
D pounds

13
Standardized Test Tutor: Math, Grade 4 © Michael Priestley, Scholastic Teaching Resources
Test 1 Name Test
Tutor
says:

19. Lucinda finds a leaf from the tree outside her house. She
measures the leaf with a centimeter ruler, as shown below.

1 2 3 4 5 6 7 8 9 10 11 12 13 14 15 16 17 18

Pay attention to where


the leaf is lined up
(at 3 centimeters, not zero).

About how long is the leaf?


A 3 centimeters
B 12 centimeters
C 15 centimeters
D 18 centimeters

20. The figure below shows the shape of Mrs. Jimenez’s garden.

10 ft

16 ft

Mrs. Jimenez wants to put a fence around the perimeter of her


garden. How long will the fence be? The garden is shaped like
a rectangle, so the opposite
A 26 feet sides are equal in length.
B 36 feet The perimeter is the sum
of all four sides.
C 52 feet
D 160 feet

21. Winnie wants to buy a rectangular piece of fabric that is 6 feet


wide and 8 feet long. What is the area of the fabric? To find the area of a
rectangle, multiply the
A 14 square feet
length by the width.
B 24 square feet
C 28 square feet
D 48 square feet

14
Standardized Test Tutor: Math, Grade 4 © Michael Priestley, Scholastic Teaching Resources
Test 1 Name Test
Tutor
says:

22. A window is divided into many panes of glass, as shown below.

What shape are the small panes of glass in the window?


Note that the window
A right triangles is divided into nine
B acute triangles rectangles, and each
rectangle is divided into
C obtuse triangles
two right triangles.
D equilateral triangles

23. In the picture below, the car’s rear window is the white part
of the drawing.

What is the shape of the rear window?


Look for the name of a
A hexagon
figure with only two sides
B rectangle that are parallel.
C pentagon
D trapezoid

15
Standardized Test Tutor: Math, Grade 4 © Michael Priestley, Scholastic Teaching Resources
Test 1 Name Test
Tutor
says:

24.
4

Which figure is similar to the triangle shown above?


Keep in mind that similar
figures have the same shape
2
but not the same size.
A 4

4
B 6

2
C 8

D 8

25. Byron receives a package that is shaped like a rectangular


prism. How many faces does the package have? Draw a picture of a
rectangular box and count
A 4 the number of faces.
B 6
C 8
D 10

16
Standardized Test Tutor: Math, Grade 4 © Michael Priestley, Scholastic Teaching Resources
Test 1 Name Test
Tutor
says:

26. Look at the grid below.


y

9
8
7
6
A B
5
4
3
2
D C
1
x
0 1 2 3 4 5 6 7 8 9

Which point on the grid is located at (5, 1)?


To find a location on a grid,
A point A count the number of units
B point B across and then the number
of units up or down.
C point C
D point D

27. Sheri draws a picture of a house on a grid. The peak of the


roof is located at (7, 8). Which figure could be the one that Look at the ordered pair.
Sheri drew? Use the first number to
count across the grid and
y y
  the second number to count
9 9 up or down.
8 8
7 7
6 6
5 5
4 4
3 3
2 2
1 1
x x
0 1 2 3 4 5 6 7 8 9 0 1 2 3 4 5 6 7 8 9

y y
 
9 9
8 8
7 7
6 6
5 5
4 4
3 3
2 2
1 1
x x
0 1 2 3 4 5 6 7 8 9 0 1 2 3 4 5 6 7 8 9

17
Standardized Test Tutor: Math, Grade 4 © Michael Priestley, Scholastic Teaching Resources
Test 1 Name Test
Tutor
says:

28. Look at the street map.

Green Street

Apple Street
et
S tre
Pine

West Street

Which two streets on the map appear to be parallel?


Remember that parallel
A Green Street and West Street lines do not intersect.
B Pine Street and Apple Street
C Pine Street and West Street
D Green Street and Apple Street

29. The figure shows a pair of glasses from the front.

How many lines of symmetry does the figure have from


this view? Use your pencil to draw
a line down the middle
A 0 of this figure and a line
B 1 across the middle of this
figure. If the figure has the
C 2 same shape on both sides
D 4 of the line, then it has a line
of symmetry.

18
Standardized Test Tutor: Math, Grade 4 © Michael Priestley, Scholastic Teaching Resources
Test 1 Name Test
Tutor
says:

30. Imran finds an old trunk in the attic. The figure below shows
the size of the trunk.

1.5 ft

2 ft
3 ft

What is the volume of the trunk?


To find volume, multiply
A 4.5 cubic feet
the length by the width
B 6.5 cubic feet by the height.
C 9 cubic feet
D 10.5 cubic feet

31. Andrea left for school at 7:44 a.m. She arrived at 8:02 a.m.
How long did it take her to get to school? First, keep in mind that
there are 60 minutes in
A 18 minutes an hour. You’ll need to
B 28 minutes write two number sentences
to find the difference
C 42 minutes
between the starting time
D 58 minutes and the ending time.

19
Standardized Test Tutor: Math, Grade 4 © Michael Priestley, Scholastic Teaching Resources
Test 1 Name Test
Tutor
says:

32. Look at the figure on the grid.


y

9
8
7
6
5
4
3
2
1
x
0 1 2 3 4 5 6 7 8 9

Which picture shows a reflection of this figure?


y y A reflection is a “flip,” or
  a mirror image. Look at the
9 9
8 8 answer choices. Do you see
7 7 a figure that points in the
6 6 opposite direction?
5 5
4 4
3 3
2 2
1 1
x x
0 1 2 3 4 5 6 7 8 9 0 1 2 3 4 5 6 7 8 9

y y
 
9 9
8 8
7 7
6 6
5 5
4 4
3 3
2 2
1 1
x x
0 1 2 3 4 5 6 7 8 9 0 1 2 3 4 5 6 7 8 9

20
Standardized Test Tutor: Math, Grade 4 © Michael Priestley, Scholastic Teaching Resources
Test 1 Name Test
Tutor
says:

33. Which figure could be folded to form a rectangular prism like


the one shown below? Picture in your mind what
this figure would look like
if you folded each side
down flat. Then find the
picture that matches.

 

 

34. At the movie theater, Mr. Oscar made a graph to show how
many tubs of popcorn he sold each day. Look at the graph.
Popcorn Sales

Monday

Tuesday

Wednesday

Thursday

Friday

Key:  20 tubs of popcorn

How many tubs of popcorn did Mr. Oscar sell on Wednesday?


Note the key under the
A 6 C 60 graph. Each symbol on
B 21 D 120 the graph stands for 20 tubs
of popcorn.

21
Standardized Test Tutor: Math, Grade 4 © Michael Priestley, Scholastic Teaching Resources
Test 1 Name Test
Tutor
says:

35. The graph below shows the kinds of birds Angelo sees at
his birdfeeder.
Blue jays

Cardinals

Sparrows
Chickadees

Based on the graph, which statement is true?


Read each answer choice
A Angelo saw more blue jays than cardinals and chickadees carefully and compare the
combined. statement to what you see
B More chickadees visited Angelo’s birdfeeder than sparrows. on the graph.

C Angelo saw more sparrows than all the other kinds of birds
combined.
D About the same number of chickadees and blue jays visited
Angelo’s birdfeeder.

36. Justine tosses a coin three times. Each time, the coin lands on
heads (H) or tails (T). The tree diagram shows the possible
outcomes of Justine’s coin tosses.

H T

H T H T

H T H T H T H T

Which list shows all of the possible outcomes?


Remember that
A {HT, HT, HT, HT} each possible outcome
B {HT, HTHT, HTHTHTHT} is represented by one path
from the top of the tree to
C {HHH, HTT, THH, TTT}
the bottom.
D {HHH, HHT, HTH, HTT, THH, THT, TTH, TTT}

22
Standardized Test Tutor: Math, Grade 4 © Michael Priestley, Scholastic Teaching Resources
Test 1 Name Test
Tutor
says:

37. Danny has 3 yellow socks, 9 green socks, 6 blue socks,


and 5 white socks in a drawer. If he reaches into the drawer There are four colors of
without looking and takes out one sock, what color is he most socks in the drawer. Which
likely to get? color has the largest number
of socks?
A yellow
B green
C blue
D white

38. Five friends have a contest to see how long they can swim
underwater. Their results are shown in the table below.

Name Time (seconds)

Jody 22

Nayem 12

Carey 15

Miguel 19

Isabel 12

What is the mean of their times?


Don’t forget that the mean
A 10 seconds is the average. Find the sum
B 12 seconds of all the times and divide
by the number of times.
C 15 seconds
D 16 seconds

23
Standardized Test Tutor: Math, Grade 4 © Michael Priestley, Scholastic Teaching Resources
Test 1 Name Test
Tutor
says:

39. The chart below shows the prices of fruits and vegetables at
the Rainbow Market. Note that the range is
the difference between
Rainbow Market Prices the highest and the
lowest numbers on the
Fruit/Vegetable Price (per pound) chart. Subtract.

Apples $1.59

Pears $2.10

Carrots $0.70

Broccoli $1.89

Squash $1.89

What is the range of the prices at Rainbow Market?


A $0.30 C $1.89
B $1.40 D $2.10

40. Mimi finds that 45% of her classmates have black hair, 35%
have brown hair, 15% have blonde hair, and 5% have red hair. First, find the graph
Which circle graph best shows Mimi’s data? that shows 45 percent
“black.” Then check the
 Hair Colors  Hair Colors other numbers to find the
correct graph.
Red
Red
Blonde Brown
Black
Black
Brown
Blonde

 Hair Colors  Hair Colors


Red Red

Blonde Blonde
Black Black
Brown
Brown

24
Standardized Test Tutor: Math, Grade 4 © Michael Priestley, Scholastic Teaching Resources
Test 1 Name Test
Tutor
says:

41. The picture shows a pattern of shapes.

If the pattern continues, what will be the next figure?


Note that the pattern
uses only three shapes.
A C Pay attention to how
the shapes turn.

B D

42. Which number sentence is equal to 30  3x  120?


Look at each answer choice
A 3x  30  120 to find the number sentence
B 3(30  x)  120 that means the same.

C 3x(30)  120
D 30  3x  120

43. Miwa wants to buy at least 100 baseball cards. A store sells
baseball cards in packs of 8. Which inequality shows the Write a number sentence
number of packs of baseball cards (p) she must buy to have to represent the situation.
Then look at the answer
at least 100 cards?
choices to find your number
A 8p  100 sentence.
B p  100  8
C 8p  100
D p  8  100

25
Standardized Test Tutor: Math, Grade 4 © Michael Priestley, Scholastic Teaching Resources
Test 1 Name Test
Tutor
says:

44. Darrell is doing a walk for charity. For every mile he walks,
his parents will donate $5 to the charity. Which table shows Every mile Daniel walks,
the relationship between the number of miles Darrell walks he earns $5, or 1  $5.
and the amount of money his parents donate? How much will he earn for
2 miles? For 3 miles?
A Number of miles 2 4 6
Amount ($) 5 10 15

B Number of miles 1 3 5
Amount ($) 5 8 10

C Number of miles 2 3 4
Amount ($) 7 8 9

D Number of miles 1 2 4
Amount ($) 5 10 20

45. There are 119 rows of seats at a baseball field. Each row has 9,
10, or 11 seats. Which is the most reasonable estimate of the To find an estimate,
total number of seats at the field? remember to round each
number to the nearest ten
A 120 and then multiply.
B 1,200
C 12,000
D 120,000

End of Test 1 STOP

26
Standardized Test Tutor: Math, Grade 4 © Michael Priestley, Scholastic Teaching Resources
Test 2 Name Date

Directions: Read each question. Look at the Test Tutor’s tip for
answering the question. Then find the answer. You may do your
work on this page or on scrap paper. Mark your answer by filling Test
in the correct bubble. Tutor
says:

1. The distance between the earth and the moon is three


hundred eighty-four thousand four hundred kilometers.
What is this number in standard form? Read each answer choice to
A 3,844 yourself to see what number
it represents.
B 384,400
C 384,400,000
D 384,000,400

2. What fraction is represented by point R on the number line?


To compare measurements,
R
count the hash marks on the
0 1
number line to find what
they represent.
A ​ __78 ​
B ​ __34 ​
C ​ __12 ​
D ​ __58 

27
Standardized Test Tutor: Math, Grade 4 © Michael Priestley, Scholastic Teaching Resources
Test 2 Name Test
Tutor
says:

 ​ 3. Students in Mr. Osborne’s class are measuring bean plants.


Each group reports the height of its plant in a different way,
as shown in the table.

Height of bean plant


Group
(inches)
1 2
9
2 ​ __
4
 ​

3 0.8
3
4 ​ ___  ​ 
10

Which group had the tallest bean plant?


Change all of the heights to
A Group 1 decimals or to like fractions.
B Group 2
C Group 3
D Group 4

​ 35
4. June runs __8
  ​miles. What is another way to write this number?
Divide the numerator by
A 3 __​ 58 ​ the denominator.
B ​ 38 ​
4 __

4 __ 3  ​
C ​ 35

8 __ 5  ​
D ​ 35

5. In 2007 the population of Laos was 6,521,998. What digit in this
number is in the hundred thousands place? Write this number in
expanded form to show
A 1 the value of each digit.
B 2
C 5
D 9

28
Standardized Test Tutor: Math, Grade 4 © Michael Priestley, Scholastic Teaching Resources
Test 2 Name Test
Tutor
says:

6. A store sells juice boxes in packages of 8. Mr. Chowdhury


bought a number of packages of juice boxes. Which could Look for a number that is
be the total number of juice boxes he bought? a multiple of 8.
A 56
B 68
C 76
D 92

7. Movie tickets at the theater cost $9 each. Total ticket sales for
one movie at the theater were $3,276. How many tickets did the Think about which
theater sell for the movie? operation can be used
to split the total sale by
A 338 individual ticket prices.
B 364
C 3,285
D 29,484

8. Kevin ate __​ 25 ​of a pie. Su Yang ate __3  ​of the pie. How much of
​ 10
the pie did Kevin and Su Yang eat together? Find a common denominator
for these two fractions
5  ​
A ​ __
15 before you add the fractions.

B ​ __35 ​
6  ​
C ​ __
50
7  ​
D ​ __
10

9. Kerby’s backpack weighs 3.56 kilograms. Bart’s backpack weighs


2.6 kilograms. How much heavier is Kerby’s backpack than Consider what kind of
Bart’s? number sentence you can
write to find the difference.
A 0.96 kilograms
B 1.5 kilograms
C 1.96 kilograms
D 3.3 kilograms

29
Standardized Test Tutor: Math, Grade 4 © Michael Priestley, Scholastic Teaching Resources
Test 2 Name Test
Tutor
says:

10. The temperature in Alvin’s freezer is 2C. The temperature in


his refrigerator is 3C. How much warmer is the temperature in Look for key words to help
the refrigerator than in the freezer? you solve the problem. In
this question, the key words
A 1C
are how much warmer.
B 3C
C 5C
D 6C

11. Toni buys an apple for $0.98, a granola bar for $1.39, and a drink
for $1.52. About how much money does she spend? Round each item’s
price before you find
A $3.00 the answer.
B $3.50
C $3.70
D $3.90

12. Sharon has 50 posters to put up for a concert. She puts up 23 in


her school. She writes a number sentence to show how many
posters (p) she has left: 50  23  p.

Which number sentence is equal to the one Sharon wrote?


Solve the number sentence
A 50  23  p to find the value of p. Then
B p  23  50 check to see which answer
option makes sense.
C p  23  50
D 23 3 p  50

13. Lana buys 4 shirts for $24.80 each. She has a coupon for $5.95
off her total purchase. What is the total amount Lana spends Remember to subtract
after using her coupon? the value of the coupon.

A $18.85
B $75.40
C $93.25
D $99.20

30
Standardized Test Tutor: Math, Grade 4 © Michael Priestley, Scholastic Teaching Resources
Test 2 Name Test
Tutor
says:

14. Little Horse Elementary School has a field day at the end of
the year. All 288 students are divided evenly into 12 teams. Write a number sentence to
On each team, __ ​ 23 ​of the students will play in the kickball solve this problem.
tournament. How many students on each team will play
in the kickball tournament?
A 8
B 16
C 18
D 24

15. Find the value of m in the number sentence below.


Try each answer choice in
7  m  21 the number sentence to find
the correct value.
A m3
B m7
C m  14
D m  28

16. A road race began at Time 1 and ended at Time 2.


Time 1 Time 2

11 12 1 11 12 1
10 2 10 2
9 3 9 3
8 4 8 4
7 6 5 7 6 5

How long did the race last?


Write down the times on
A 5 minutes the clocks to help you find
B 5 hours the answer.

C 1 hour 5 minutes
D 1 hour 12 minutes

31
Standardized Test Tutor: Math, Grade 4 © Michael Priestley, Scholastic Teaching Resources
Test 2 Name Test
Tutor
says:

17. Micah’s house is 30 feet tall. About how many meters tall
is Micah’s house? Remember that a meter is
almost the same as a yard.
A 3 meters
B 10 meters
C 30 meters
D 90 meters

18. What unit of measurement would be best to describe the area


of a town? Look for the largest unit
of measurement.
A square kilometers
B square meters
C square feet
D square millimeters

19. Mr. Kenworthy bought some potatoes at a farmstand.

0 lb

3 lb 1 lb

2 lb

What is the weight of the potatoes shown on the scale? Count the hash marks
A 2.7 pounds between each pound mark
to help you read the weight
B 2.8 pounds
shown on the scale.
C 3.2 pounds
D 3.3 pounds

32
Standardized Test Tutor: Math, Grade 4 © Michael Priestley, Scholastic Teaching Resources
Test 2 Name Test
Tutor
says:

20. Silas wants to cover one rectangular wall of his room with
wallpaper. The wall is 8 feet tall and 13 feet long. What is the Use the formula for finding
area of the wall? area (width  length).
A 21 square feet
B 42 square feet
C 52 square feet
D 104 square feet

21. A rectangular playing field is 73 meters long and 59 meters wide.


Travis ran around the perimeter of the field once. Which is the Remember that the opposite
sides in a rectangle are
best estimate of how far Travis ran?
equal, and the perimeter is
A 120 meters the sum of all four sides.
B 130 meters
C 240 meters
D 260 meters

22. Look at the figure below.


M N

P O

Which angle in the figure is obtuse? Remember that an


A M obtuse angle is greater
than 90 degrees.
B N
C O
D P

33
Standardized Test Tutor: Math, Grade 4 © Michael Priestley, Scholastic Teaching Resources
Test 2 Name Test
Tutor
says:

23. Which of these figures is not a parallelogram?


Note that a parallelogram
has two pairs of parallel
A sides.

24. Which figure is congruent to the one below?


Congruent figures are
the same size and the
3 same shape.

6 3

A C
6

B 4 D 3

8
3

34
Standardized Test Tutor: Math, Grade 4 © Michael Priestley, Scholastic Teaching Resources
Test 2 Name Test
Tutor
says:

25. Which of these objects is a sphere?


Look for a figure that
doesn’t have any flat faces.

26. What are the coordinates of point R?


y Remember that the first
number in an ordered pair
9
8
is the number of units
7 across the grid.
6
5
4
3
R
2
1
x
0 1 2 3 4 5 6 7 8 9

A (2, 4)
B (4, 0)
C (4, 2)
D (4, 4)

35
Standardized Test Tutor: Math, Grade 4 © Michael Priestley, Scholastic Teaching Resources
Test 2 Name Test
Tutor
says:

27. Molly drew this figure on a grid.


y

9
8
7
6
Q
5
4
3
2 R
1
T S
x
0 1 2 3 4 5 6 7 8 9

Which vertex of the figure is located at (2, 5)?


Count the units across first
A point Q and then the units up.
B point R
C point S
D point T

28. Which letter shown below has perpendicular lines?


Keep in mind that
perpendicular lines form
a right angle.

A W
B X
C N
D H

29. How many lines of symmetry does this figure have?


Draw lines across and down
the middle of the figure to
find the answer.

A 0 C 2
B 1 D 4

36
Standardized Test Tutor: Math, Grade 4 © Michael Priestley, Scholastic Teaching Resources
Test 2 Name Test
Tutor
says:

30. Look at the figure on the grid.


y

9
8
7
6
5
4
3
2
1
x
0 1 2 3 4 5 6 7 8 9

Which picture shows the figure translated 2 units to the right


and 1 unit down? Choose one corner of the
y y figure to use in counting
  units to the right and down.
9 9
8 8
7 7
6 6
5 5
4 4
3 3
2 2
1 1
x x
0 1 2 3 4 5 6 7 8 9 0 1 2 3 4 5 6 7 8 9

 y
 y
9 9
8 8
7 7
6 6
5 5
4 4
3 3
2 2
1 1
x x
0 1 2 3 4 5 6 7 8 9 0 1 2 3 4 5 6 7 8 9

37
Standardized Test Tutor: Math, Grade 4 © Michael Priestley, Scholastic Teaching Resources
Test 2 Name Test
Tutor
says:

31. Judith needs to mow her lawn, which is shown as the gray area
of the figure below. The dimensions of her whole property are
10 meters by 14 meters. Her house measures 8 meters by 8 meters.

14 m
8m

8m

10 m

What is the area of lawn that Judith must mow?


Remember that the area of
A 8 square meters the lawn does not include
the house.
B 16 square meters
C 76 square meters
D 204 square meters

32. Mrs. Van is making jam. The jam needs to cook at a temperature
of 218F. Right now the temperature of the jam is 199.3F. How Choose the correct
much higher does the temperature have to rise? operation to find
the difference.
A 18.7F
B 19.7F
C 177.5F
D 417.3F

38
Standardized Test Tutor: Math, Grade 4 © Michael Priestley, Scholastic Teaching Resources
Test 2 Name Test
Tutor
says:

33.

Which figure could be folded to form a triangular pyramid like the


one shown above? Picture in your mind
what the figure would look
like with each side folded
down flat.
A C

B D

34. The chart shows ticket prices for the aquarium. A visitor may
choose to purchase any combination of tickets.

Aquarium Ticket Prices


Main Sea Otter Dolphin
Aquarium Exhibit Show

Adult $15 $5 $7

Child $8 $3 $4

Senior $10 $4 $5

Which statement about the ticket prices is true?


Read each statement
A It costs more for a senior to visit the dolphin show than carefully and use the
for a child to visit the main aquarium. numbers from the chart
B A child can visit the main aquarium, sea otter exhibit, to see if it is true.
and dolphin show for the same price as an adult ticket
to the main aquarium.
C It costs more for a senior to visit the main aquarium than
for an adult to visit the sea otter exhibit and the dolphin show.
D A senior can go to the sea otter exhibit and the dolphin show
for the same price as a child’s ticket to the main aquarium.

39
Standardized Test Tutor: Math, Grade 4 © Michael Priestley, Scholastic Teaching Resources
Test 2 Name Test
Tutor
says:

35. Betsy is taking a gymnastics class. Each month, the students in


the class see how long they can hold a handstand. Betsy keeps
track of her handstand times in a line graph.
Betsy’s Handstand Times
10
9
8
Time (in seconds)

7
6
5
4
3
2
1
0
May June July Aug Sep Oct
Month

Which statement about the line graph is true?


Make sure that you
A Betsy’s times increased steadily over the six months. understand the line graph
B Betsy’s time increased the most between May and June. before you consider the
possible answers.
C Betsy’s time was greatest in August.
D Betsy’s times were the same in June and September.

36. Nina made a spinner for a game.

J K

Which list shows all of the possible outcomes of spinning


the spinner twice? Make a list to show the
possible outcomes for one
A {JK, JL, KL} turn. Then compare it to the
B {JK, JL, KJ, KL, LJ, LK} answer choices.

C {JKL, JLK, KLJ, KJL, LJK, LKJ}


D {JJ, JK, JL, KJ, KK, KL, LJ, LK, LL}

40
Standardized Test Tutor: Math, Grade 4 © Michael Priestley, Scholastic Teaching Resources
Test 2 Name Test
Tutor
says:

37. Andrew has a bag of candies: 5 red candies, 3 blue candies,


and 2 yellow candies. If Andrew picks one candy without There are 10 candies in all.
looking, what is the probability that he will pick a blue candy? How many are blue?
3  ​
A ​ __
10
B ​ __37 ​
C ​ __12 ​
7  ​
D ​ __
10

38. Igor read four books last month. The books were 125 pages,
96 pages, 157 pages, and 278 pages long. What is the mean Remember, finding the
number of pages per book that Igor read? mean is the same as finding
the average.
A 141 pages
B 164 pages
C 182 pages
D 656 pages

39. Mr. Lund planted 20 tomato plants. He kept track of the number
of tomatoes he picked from each plant. His results are shown on
the diagram of his garden below.

8 12 6 9 7

9 7 10 14 4

8 8 11 9 5

13 10 3 4 9

What is the mode of the data Mr. Lund collected?


Remember that the mode
A 8 C 9 is the number that appears
B 8.5 D 11 most often.

41
Standardized Test Tutor: Math, Grade 4 © Michael Priestley, Scholastic Teaching Resources
Test 2 Name Test
Tutor
says:

40. Ms. Der asked her students to choose a class pet. She presented
her data in a graph, shown below.

Class Pet Survey


10
9
Number of Students

8
7
6
5
4
3
2
1
0
Fish Lizard Hamster
Type of Pet

How many more students chose a hamster than chose a fish?


Compare the heights of
A 3 the bars on the graph.

B 4
C 6
D 7

41. A number pattern begins: 3, 6, 9, 12, . . .


Which expression can be used to find the next number
in the pattern? Use the numbers you know
to find the rule for this
A 34 pattern that will determine
the next number.
B 3  12
C 53
D 12  3

42
Standardized Test Tutor: Math, Grade 4 © Michael Priestley, Scholastic Teaching Resources
Test 2 Name Test
Tutor
says:

42. Moira has $10.00 to buy some toys. Bouncy balls and toy
dinosaurs cost $0.50 each. Moira writes a number sentence
to show how many bouncy balls (b) and dinosaurs (d) she
can buy:

$0.50(b  d)  $10.00
Which number sentence is equal to the one Moira wrote?
Look at each answer choice
A $0.50b  $0.50d  $10.00 carefully before you choose.
B $0.50bd  $10.00
C $0.50b  $10.00  d
D $0.50b  d  $10.00

4
3. Mrs. Aguillera buys a package of 360 stickers. She wants
To choose the correct
to give each student in her class the same number of
operation, think about the
stickers. If there are s students in Mrs. Aguillera’s class,
concept of splitting things
which expression can she use to find the number of stickers
among friends.
each student will receive?

A 360  s
B 360  s
C 360  s
D 360  s

43
Standardized Test Tutor: Math, Grade 4 © Michael Priestley, Scholastic Teaching Resources
Test 2 Name Test
Tutor
says:

44. Mr. Longo likes to walk. For every hour he walks, he goes
2 miles. Which graph shows this relationship between the If Mr. Longo walks 2 miles in
number of hours and the number of miles he walks? 1 hour, how far will he walk
y y in 2 hours? In 3 hours?
 
9 9
8 8
7 7
6 6
5 5
4 4
3 3
2 2
1 1
x x
0 1 2 3 4 5 6 7 8 9 0 1 2 3 4 5 6 7 8 9

y y
 
9 9
8 8
7 7
6 6
5 5
4 4
3 3
2 2
1 1
x x
0 1 2 3 4 5 6 7 8 9 0 1 2 3 4 5 6 7 8 9

45. A rectangular swimming pool is 60 feet wide. Its length is


Use the numbers you know
greater than its width, but not greater than 100 feet. Which
to find an estimate for the
is the best estimate of the area of the swimming pool?
area (length  width).
A 160 square feet
B 320 square feet
C 4,800 square feet
D 7,200 square feet

End of Test 2 STOP

44
Standardized Test Tutor: Math, Grade 4 © Michael Priestley, Scholastic Teaching Resources
Test 3 Name Date

Good Luck!

Directions: Choose the best answer to each question. Mark


your answer by filling in the correct bubble.

1. A factory makes five hundred sixty thousand three


hundred twenty boxes in one month. What is this
number in standard form?

A 5,632
B 56,320
C 560,320
D 563,020

2. What number does point H represent on the number line?


H
0 1.0 2.0 3.0

A 1.8
B 2.2
C 2.4
D 2.8

3. Four students are reading the same book. The chart below
shows how much of the book each student has read.

Student Amount of book read

Sasha ​ 3   ​
___
10

José 4 ​
​ __
5

Fran 2 ​
​ __
3
5
Neal ​ __ ​
9

Who has read the least?

A Sasha C Fran
B José D Neal

45
Standardized Test Tutor: Math, Grade 4 © Michael Priestley, Scholastic Teaching Resources
Test 3 Name

4. Which fraction is equivalent to __8  ​?


​ 12

A ​ __34 ​
12  ​
B ​ __ 8
4
__
C ​ 9 ​
24 ​
D ​ __
36

5. A new stereo costs $1,653.89. Which digit in this number


is in the tenths place?

A 3
B 5
C 8
D 9

6. The table below shows the number of students in each grade


at Oak Bluff Elementary School.

Grade Number of Students

1 27

2 23

3 34

4 33

In which grade is the number of students a prime number?

A Grade 1
B Grade 2
C Grade 3
D Grade 4

46
Standardized Test Tutor: Math, Grade 4 © Michael Priestley, Scholastic Teaching Resources
Test 3 Name

7. A train has 7 cars with seats for passengers. Each car has 128
seats. How many seats are there all together?
A 135
B 746
C 796
D 896

8. Tim bought __​ 13 ​pound of rice, and Rita bought __5  ​pound of rice.


​ 12
How many more pounds of rice did Rita buy than Tim?

1  ​pound
A ​ __
12

B ​ __16 ​pound
C ​ __13 ​pound
D ​ __49 ​pound

9. Isabel buys a shirt for $25.95, socks for $7.39, and a hat
for $12.60. What is the total amount she spends?

A $33.34
B $34.60
C $38.55
D $45.94

47
Standardized Test Tutor: Math, Grade 4 © Michael Priestley, Scholastic Teaching Resources
Test 3 Name

10. In a board game, each player draws two cards on a turn.
The sum of the numbers on the cards is the number of spaces
the player moves. Nicole draws the two cards shown below.

How many spaces does Nicole move?

A 1 C 4
B 3 D 5

11. Ned weighs 98 pounds, his brother weighs 111 pounds, and
his father weighs 169 pounds. What is the best estimate of how
much all three of them weigh together?

A 300 pounds
B 360 pounds
C 380 pounds
D 400 pounds

12. Charlton wants to practice the piano for 45 minutes. He has


already practiced for 24 minutes. He writes a number sentence
to show how many more minutes (m) he needs to practice:

24  m  45
Which number sentence is equivalent to the one Charlton wrote?
A 24  m  45
B 45  m  24
C 45  m  24
D m  24  45

48
Standardized Test Tutor: Math, Grade 4 © Michael Priestley, Scholastic Teaching Resources
Test 3 Name

13. Ozzie buys 6 packages that each contain 12 balloons. He


gives each of the 9 guests at his party the same number
of balloons. He has none left over. How many balloons
does each guest receive?
A 2
B 8
C 9
D 27

14. Each school day, Cherry walks 1.8 kilometers to her friend’s
house and then rides her bike for 5.6 kilometers to school.
On the way home, she does the same thing in reverse. How
far does Cherry travel going to school and back each day?
A 6.4 kilometers
B 7.4 kilometers
C 12.8 kilometers
D 14.8 kilometers

15. Find the value of n in the number sentence below.


n  6  18
A n3
B n6
C n  12
D n  24

49
Standardized Test Tutor: Math, Grade 4 © Michael Priestley, Scholastic Teaching Resources
Test 3 Name

16. The clocks below show the time at the start of a movie and
at the end.
Start End

How long did the movie last?


A 1 hour 17 minutes
B 1 hour 57 minutes
C 2 hours 17 minutes
D 2 hours 57 minutes

17. Gary’s soup recipe calls for 10 cups of broth. How many quarts
of broth does he need?
A ​ 12 ​
2 __
B 5
C 6
D 40

18. Malee wants to measure the length of her shoe. What is the best
unit of measurement for her to use?
A meters
B centimeters
C millimeters
D kilometers

50
Standardized Test Tutor: Math, Grade 4 © Michael Priestley, Scholastic Teaching Resources
Test 3 Name

19. Jared picks a carrot from the garden and measures its length
with a ruler.

1 2 3 4 5 6

How long is the carrot (including the stem)?


A 4 inches

B ​ 12 ​inches
4 __

C 5 inches

D ​ 12 ​inches
5 __

20. Haley is fencing off a triangular play space for her dog,
as shown below.

x
24 ft

32 ft

Haley uses a total of 96 feet of fencing for the perimeter of the


triangle. What is the length of the third side of the triangle (x)?
A 40 feet
B 50 feet
C 56 feet
D 64 feet

51
Standardized Test Tutor: Math, Grade 4 © Michael Priestley, Scholastic Teaching Resources
Test 3 Name

21. A rectangular field is 100 meters long and 20 meters wide. What
is the area of the field?
A 120 square meters
B 200 square meters
C 240 square meters
D 2,000 square meters

22. Which triangle is an acute triangle?

A C

B D

23.

The bricks in a brick wall are shaped like rectangles. What must
be true of each brick?
A Each brick has two pairs of parallel sides.
B Each brick has four equal sides.
C Each brick has four acute angles.
D Each brick has two obtuse angles and two acute angles.

52
Standardized Test Tutor: Math, Grade 4 © Michael Priestley, Scholastic Teaching Resources
Test 3 Name

24. Jee made this drawing of his front door.

2m

1m

Which figure is similar to Jee’s drawing?

A 4m C 6m

1m
3m

B 8m D 10 m

2m
4m

25. Which figure has exactly five faces?


A cube
B sphere
C rectangular sphere
D square pyramid

53
Standardized Test Tutor: Math, Grade 4 © Michael Priestley, Scholastic Teaching Resources
Test 3 Name

26. What are the coordinates of point F on the grid?


y

9
8
7
F
6
5
4
3
2
1
x
0 1 2 3 4 5 6 7 8 9

A (6, 4)
B (4, 6)
C (6, 6)
D (4, 4)

27. Which vertex of the square is located at (2, 7)?


y

9
8
P Q
7
6
5
4
3
2
S R
1
x
0 1 2 3 4 5 6 7 8 9

A P
B Q
C R
D S

54
Standardized Test Tutor: Math, Grade 4 © Michael Priestley, Scholastic Teaching Resources
Test 3 Name

28. Which circle contains parallel lines?


 

 

29. Which figure has exactly two lines of symmetry?

55
Standardized Test Tutor: Math, Grade 4 © Michael Priestley, Scholastic Teaching Resources
Test 3 Name

30. Look at the figure on the grid.


y

9
8
7
6
5
4
3
2
1
x
0 1 2 3 4 5 6 7 8 9

Which picture shows a rotation of this figure?

y y
 
9 9
8 8
7 7
6 6
5 5
4 4
3 3
2 2
1 1
x x
0 1 2 3 4 5 6 7 8 9 0 1 2 3 4 5 6 7 8 9

y y
 
9 9
8 8
7 7
6 6
5 5
4 4
3 3
2 2
1 1
x x
0 1 2 3 4 5 6 7 8 9 0 1 2 3 4 5 6 7 8 9

56
Standardized Test Tutor: Math, Grade 4 © Michael Priestley, Scholastic Teaching Resources
Test 3 Name

31. Daniel has a rock and a stick. When he puts the rock on a scale,
the scale reads 2.4 pounds. When he puts both the rock and the
stick on the scale, the scale reads 3.3 pounds. How much does
the stick weigh?
A 0.9 pound
B 1.9 pounds
C 3.3 pounds
D 5.7 pounds

32. A rectangular parking lot is 50 meters long and 30 meters wide.


If a policewoman walks the entire perimeter of the lot, how far
has she walked?
A 80 meters
B 130 meters
C 160 meters
D 1,500 meters

33. Which figure could be folded to form a cube like the one
shown below?

A C

B D

57
Standardized Test Tutor: Math, Grade 4 © Michael Priestley, Scholastic Teaching Resources
Test 3 Name

34. Mira takes a survey of her classmates to see how they get
to school. The graph below shows her results.
Mira’s
Transportation Graph
10
9
Number of Classmates

8
7
6
5
4
3
2
1
0
Walk Bus Drive Bike
How Classmates Get to School

Which statement about the results of Mira’s survey is true?


A More classmates take the bus than walk and bike combined.
B More classmates walk than drive and bike combined.
C Most classmates walk to school.
D The fewest classmates drive to school.

58
Standardized Test Tutor: Math, Grade 4 © Michael Priestley, Scholastic Teaching Resources
Test 3 Name

35. Fernando earns money for weeding his mother’s garden.


The graph below shows how much he got paid each week.
Fernando’s Earnings
20
18
16
Money Earned ($)

14
Amount of

12
10
8
6
4
2
0
1 2 3 4 5 6
Week

Based on the graph, which statement is true?


A The amount of money Fernando earned steadily decreased.
B Fernando earned the greatest amount of money in week 3.
C The amount Fernando earned dropped the most between
weeks 5 and 6.
D Fernando earned the least amount of money in week 4.

59
Standardized Test Tutor: Math, Grade 4 © Michael Priestley, Scholastic Teaching Resources
Test 3 Name

36. The menu from Pancake Palace is shown below.

Pancake Palace
Pancakes Toppings
Regular (R) Syryp (S)
Whole wheat (W) Bananas (B)
Nuts (N)
Powdered sugar (P)

Dennis wants to choose one kind of pancake and one topping.


Which list shows all of the possible combinations he can choose?
A {RS, WB, NP}
B {RW, SBNP}
C {RS, RB, RN, RP, WS, WB, WN, WP}
D {RW, RS, RB, RN, RP, WS, WB, WN, WP, SB, SN, SP, BN, NP, BN}

37. Clay made this spinner for a game.

Red Green

Blue Yellow

Red Red

Blue Green

On one spin, what is the probability that the arrow will land on Blue?
A ​ __18 ​
B ​ __14 ​
C ​ __38 ​
D ​ __12 ​

60
Standardized Test Tutor: Math, Grade 4 © Michael Priestley, Scholastic Teaching Resources
Test 3 Name

38. Julio has five dogs. Their weights are shown in the table below.

Julio’s Dogs
Name Weight (lb)

Fifi 38

Flipper 82

Fido 54

Floppy 23

Fluffy 48

What is the median weight of Julio’s dogs?


A 48 lb
B 49 lb
C 59 lb
D 245 lb

39. Grace records the temperature every morning for a week.

Morning Temperature (F)


Mon Tues Wed Thu Fri Sat Sun

16 14 21 28 32 30 27

What is the range of temperatures Grace recorded?


A 11F
B 16F
C 18F
D 168F

61
Standardized Test Tutor: Math, Grade 4 © Michael Priestley, Scholastic Teaching Resources
Test 3 Name

40. Mr. Angelo asked his students to name their favorite writing
utensils. The graph below shows the results.

1
Pen —
3 1
Pencil —
2

Marker
1

6

There are 24 students in Mr. Angelo’s class. How many students


chose pencils as their favorite writing utensil?
A 2
B 4
C 8
D 12

41. A pattern begins 2, 4, 8, 16, . . .


Which expression can be used to find the next number
in the pattern?
A 16  2
B 16 3 2
C 16  8
D 16 3 4

62
Standardized Test Tutor: Math, Grade 4 © Michael Priestley, Scholastic Teaching Resources
Test 3 Name

42. Cynthia needs $50 to buy a new skateboard. She got $25
for her birthday and $5 for mowing the lawn. She writes
a number sentence to show how much money (m) she
still needs:

($25  $5)  m  $50


Which number sentence is equivalent to the one
Cynthia wrote?
A ($25  $5)  $50  m
B $25  ($5  m)  $50
C m($25  $5)  $50
D $25  ($5  m)  $50

43. Terry has a bag of crackers. After he gives 9 crackers to Diana,


he has 14 crackers left. Which equation shows how many
crackers (c) Terry started with?

A c  9  14
B 14  9  c
C c  9  14
D 14  c  9

63
Standardized Test Tutor: Math, Grade 4 © Michael Priestley, Scholastic Teaching Resources
Test 3 Name

44. A phone company charges $0.50 per minute to make a long-


distance call. Which graph shows the relationship between
the number of minutes in a phone call and the cost?
y y
 
9 9
8 8
7 7
Cost ($)

Cost ($)
6 6
5 5
4 4
3 3
2 2
1 1
x x
0 1 2 3 4 5 6 7 8 9 0 1 2 3 4 5 6 7 8 9
Minutes Minutes

y y
 
9 9
8 8
7 7
Cost ($)

Cost ($)

6 6
5 5
4 4
3 3
2 2
1 1
x x
0 1 2 3 4 5 6 7 8 9 0 1 2 3 4 5 6 7 8 9
Minutes Minutes

45. Max has to figure out the total cost of 62 tickets for a baseball
game. Each ticket costs $8.95. Which is the most reasonable
estimate of the total cost?
A $5,000
B $540
C $300
D $70

End of Test 3 STOP

64
Standardized Test Tutor: Math, Grade 4 © Michael Priestley, Scholastic Teaching Resources
Standardized Test Tutor: Math
Answer Sheet
Grade 4
Student Name Test 1 2 3
Teacher Name (circle one)

Directions: Fill in the bubble for the answer you choose.

1. 𝖠 𝖡 𝖢 𝖣 16. 𝖠 𝖡 𝖢 𝖣 31. 𝖠 𝖡 𝖢 𝖣

2. 𝖠 𝖡 𝖢 𝖣 17. 𝖠 𝖡 𝖢 𝖣 32. 𝖠 𝖡 𝖢 𝖣

3. 𝖠 𝖡 𝖢 𝖣 18. 𝖠 𝖡 𝖢 𝖣 33. 𝖠 𝖡 𝖢 𝖣

4. 𝖠 𝖡 𝖢 𝖣 19. 𝖠 𝖡 𝖢 𝖣 34. 𝖠 𝖡 𝖢 𝖣

5. 𝖠 𝖡 𝖢 𝖣 20. 𝖠 𝖡 𝖢 𝖣 35. 𝖠 𝖡 𝖢 𝖣

6. 𝖠 𝖡 𝖢 𝖣 21. 𝖠 𝖡 𝖢 𝖣 36. 𝖠 𝖡 𝖢 𝖣

7. 𝖠 𝖡 𝖢 𝖣 22. 𝖠 𝖡 𝖢 𝖣 37. 𝖠 𝖡 𝖢 𝖣

8. 𝖠 𝖡 𝖢 𝖣 23. 𝖠 𝖡 𝖢 𝖣 38. 𝖠 𝖡 𝖢 𝖣

9. 𝖠 𝖡 𝖢 𝖣 24. 𝖠 𝖡 𝖢 𝖣 39. 𝖠 𝖡 𝖢 𝖣

10. 𝖠 𝖡 𝖢 𝖣 25. 𝖠 𝖡 𝖢 𝖣 40. 𝖠 𝖡 𝖢 𝖣

11. 𝖠 𝖡 𝖢 𝖣 26. 𝖠 𝖡 𝖢 𝖣 41. 𝖠 𝖡 𝖢 𝖣

12. 𝖠 𝖡 𝖢 𝖣 27. 𝖠 𝖡 𝖢 𝖣 42. 𝖠 𝖡 𝖢 𝖣

13. 𝖠 𝖡 𝖢 𝖣 28. 𝖠 𝖡 𝖢 𝖣 43. 𝖠 𝖡 𝖢 𝖣

14. 𝖠 𝖡 𝖢 𝖣 29. 𝖠 𝖡 𝖢 𝖣 44. 𝖠 𝖡 𝖢 𝖣

15. 𝖠 𝖡 𝖢 𝖣 30. 𝖠 𝖡 𝖢 𝖣 45. 𝖠 𝖡 𝖢 𝖣

65
Standardized Test Tutor: Math, Grade 4 © Michael Priestley, Scholastic Teaching Resources
Standardized Test Tutor: Math, Grade 4 © Michael Priestley, Scholastic Teaching Resources
Test 1 Answer Key

1. D 10. C 19. B 28. D 37. B

2. B 11. B 20. C 29. B 38. D

3. A 12. C 21. D 30. C 39. B

4. D 13. D 22. A 31. A 40. A

5. A 14. C 23. D 32. C 41. C

6. B 15. C 24. A 33. D 42. A

7. A 16. A 25. B 34. D 43. C

8. D 17. B 26. C 35. C 44. D

9. C 18. D 27. C 36. D 45. B

Answer Key Explanations


1. Correct response: D 2. (continued)
(Read and write whole numbers and decimals) Incorrect choices:
The standard form of eight million two 1  ​, which is less than __
A Jerome ate ​ __ ​ 13 ​.
hundred fifty thousand has an 8 in the millions 12
​ 14 ​, which is less than __
C Charlotte ate __ ​ 13 ​.
place (8,000,000), a 2 in the hundred thousands
place (200,000), and a 5 in the ten thousands ​ 16 ​, which is less than __
D Bruce ate __ ​ 13 ​.
place (50,000): 8,250,000.
3. Correct response: A
Incorrect choices: (Compare and order whole numbers, fractions,
A is eight thousand two hundred fifty. and decimals)
On this number line, the space between
B is eight hundred twenty-five thousand.
0 and 1 is divided into five units, or fifths. Point
C is eight million two hundred fifty. M is located between the first and second hash
marks, which represent __ ​ 15 ​and __
​ 25 ​, respectively.
2. Correct response: B 1
__ __ 2 2
__ __4
​ 5 ​ ​ 10  ​ 0.2, and ​ 5 ​ ​ 10  ​ 0.4, so Point M is
(Compare and order whole numbers, fractions, between 0.2 and 0.4.
and decimals)
Since all of the fractions have 1 in the Incorrect choices:
numerator, the largest fraction is the one with B reflects an error in reading the number
the smallest number in the denominator. The line as being divided into fourths (0.25,
​ 13 ​, so Sheena
largest fraction listed in the table is __ 0.5, 0.75).
ate the most pizza.
C reflects an error in reading the number line
as being divided into tenths (0.1, 0.2, 0.3 . . .).
D reflects an error in reading the number line
as being divided into ones (0, 1, 2 . . .).

Standardized Test Tutor: Math, Grade 4 © Michael Priestley, Scholastic Teaching Resources
67
4. Correct response: D 7. Correct response: A
(Represent, convert, and find equivalent fractions (Add, subtract, multiply, and divide whole
and mixed numbers) numbers)
To find the answer, you must convert the To find the number of cans of soup sold,
mixed number to an improper fraction: multiply subtract the number of cans that were left on
the denominator (5) by the whole number (6) Saturday (6,529) from the number of cans on
and add the numerator (2): 6 __ ​ 32
​ 25 ​ __ 5
  ​. Monday (8,120): 8,120  6,529  1,591.

Incorrect choices: Incorrect choices:


A is the result of multiplying 6 by the numer- B reflects an error in subtraction (forgetting
ator 2 and dividing by the denominator 5. to rename the 1 in the hundreds place).
B is the result of multiplying 6 by the numer- C reflects an error in subtraction (forgetting
ator 2, adding 5, and dividing by 5. to “carry” to the tens place and forgetting
C is the result of multiplying 6 by the denom- to rename the 8 in the thousands place).
inator 5 and dividing by 5. D is the result of adding 8,120  6,529.

5. Correct response: A 8. Correct response: D


(Identify and use place value) (Add and subtract fractions and mixed numbers)
This number can be written in expanded To add the mixed numbers, give the fractions
form as 600,000  20,000  8,000  400  a common denominator: 2 __ ​ 34 ​ 3 __ ​ 34 ​ 3 __
​ 12 ​ 2 __ ​ 24 ​.
50  1. Ten thousand is 1 followed by 4 zeros, Add the whole numbers together (2  3  5), and
so the digit 2 is in the ten thousands place. add the fractions together to get __ ​ 54 ​, or 1 __ ​ 14 ​. 5 + 1 __ ​ 14 ​
1
__
 6 ​ 4 ​lb.
Incorrect choices:
In B the 5 is in the tens place. Incorrect choices:
In C the 6 is in the hundred thousands place. A is the result of forgetting to add the 1 in 1 ​ __14 ​
to the 5.
In D the 8 is in the thousands place.
B is the result of adding the fractions
incorrectly: ​ __34 ​ __
​ 12 ​ __
​ 46 ​ __
​ 23 ​.
6. Correct response: B
(Identify prime numbers, factors, and multiples) C is the result of converting the fractions to a
To find the answer, you must find the common denominator incorrectly: ​ __12 ​ __
​ 14 ​.
number that is a factor of 81, or a number it can
be divided by without a remainder. 3 is a factor 9. Correct response: C
of 81 (81  3  27), so the students could be (Add, subtract, multiply, and divide decimals)
divided into 3 groups with no students left over. To find the height of the stack of blocks,
multiply the number of blocks by the height
Incorrect choices: of each block: 8  6.4  51.2.
A is incorrect because 2 is not a factor of 81
(81  2  40 R1, or one student left over). Incorrect choices:
C is incorrect because 7 is not a factor of 81 A is the result of forgetting to carry the
(81  7  11 R4). 3 when multiplying.
D is incorrect because 11 is not a factor 81 B is the result of forgetting to carry the
(81  11  7 R4). 3 when multiplying and misplacing
the decimal.
D is the result of misplacing the decimal.

68 Standardized Test Tutor: Math, Grade 4 © Michael Priestley, Scholastic Teaching Resources
10. Correct response: C 13. Correct response: D
(Solve problems involving integers) (Solve multi-step problems involving whole
The amount that the temperature rose can numbers, fractions, decimals)
be found by subtracting the temperature in the The total number of fish in the pet store can
morning from the temperature in the afternoon: be found by multiplying the number of tanks by
4C (1C)  4  1  5C. the number of fish in each tank: 30  6  180.
​ 13 ​of the fish are goldfish, you can multiply
Since __
Incorrect choices:
​ 13 ​ 60; 60 fish are goldfish.
180  __
A is the result of subtracting: 4C  1C  3C.
B is the temperature in the afternoon. Incorrect choices:
D is the result of adding the zero on the A is the result of multiplying the number of
thermometer as a degree (4, 3, 2, 1, 0, fish in one tank (30) by ​ __13 ​.
and 1 appear to be 6 degrees). B is the result of adding the number of fish in
one tank to the number of tanks (30  6)
11. Correct response: B and then multiplying by __​ 13 ​.
(Estimate using whole numbers and decimals)
C is the result of multiplying the number of
To estimate the amount of time it will take
​ 13 ​and then adding
fish in one tank (30) by __
Latoya to make the cards, first round the num-
the number of tanks (6).
bers: 28 minutes can be rounded to 30 minutes,
​ 12 ​hour, and 32 people can be rounded to 30
or __ 14. Correct response: C
people. 30  __ ​ 30
​ 12 ​hour  __ 2
  ​ 15 hours. (Solve multi-step problems involving whole
numbers, fractions, decimals)
Incorrect choices: First, find the price of 10 fruit bars by
A is the result of rounding 28 minutes to 20 multiplying the number of fruit bars by the price
(30  20  600 minutes, or 10 hours). of each bar: 10  $1.25  $12.50. Next, find the
price of 12 muffins in the same way: 12  $1.40
C is the result of thinking that 30 minutes
 $16.80. Finally, add the two prices together
is about an hour (30  1  30 hours).
to get the total cost: $12.50  $16.80  $29.30.
D is the result of rounding both numbers
to 30 and adding: 30  30  60 hours. Incorrect choices:
A is the result of adding 10  $1.25 
12. Correct response: C 12  $1.40.
(Apply the properties of operations) B is the result of multiplying the wrong
Multiplication can be seen as repeated prices: (12  $1.25)  (10  $1.40).
addition. The multiplication in the number
D is the result of adding the prices together,
sentence 4  10  c can be expressed as
adding the number of fruit bars and
addition by adding 4 10’s together: 10  10  10
muffins together, and then multiplying
 10  c.
the two sums ($1.25  $1.40)  (10  12).
Incorrect choices:
A Adding two numbers does not give the
same result as multiplying them.
B would be correct if it showed the addition
of 10 4’s, but there are only 4.
D reflects a misunderstanding of the
properties of operations.

Standardized Test Tutor: Math, Grade 4 © Michael Priestley, Scholastic Teaching Resources
69
15. Correct response: C 18. Correct response: D
(Solve number sentences with one variable) (Select appropriate unit for measuring weight/
To solve the number sentence, first add x to mass, capacity, length, perimeter, and area)
both sides so that the x value is positive: Most dogs weigh between 5 and 100 pounds,
so pounds are a good measure of a dog’s weight;
15  x  x  5  x
the numbers are whole numbers and are not
15  5  x over 1,000.
Then subtract 5 from each side of the equation:
Incorrect choices:
15  5  5  x  5
A is too large a unit of measurement,
10  x since 1 ton  2,000 pounds.
Incorrect choices: B is too small a unit of measurement;
1 pound  16 ounces, and dogs do
A is the result of dividing 15 by 5 in the
not weigh less than 1 pound.
second step.
C is too small a unit of measurement (smaller
B is the result of misreading the equation
than an ounce), and it denotes mass rather
or ignoring the 15: 15  x  5.
than weight.
D is the result of adding 5 to 15 in the
second step. 19. Correct response: B
16. Correct response: A (Use rulers and other instruments to measure
(Tell time and find elapsed time) accurately)
The analog clock shows a time of 1:15, so the The leaf is lined up to the 3-cm hash mark
digital clock should read 1:15. on one end and the 15-cm hash mark on the
other end. The length of the leaf is 15 cm  3 cm
Incorrect choices:  12 cm.
B is the result of thinking that the minute
Incorrect choices:
hand pointing at the 3 means 30 minutes.
A reflects the hash mark on the ruler at one
C is the result of confusing the minute hand
end of the leaf.
(3:00) and the hour hand (:07).
C reflects the hash mark on the ruler where
D confuses the minute hand and the
the leaf ends.
hour hand and mistakes the 1 to mean
10 minutes. D is the last hash mark shown on the ruler
and reflects the length of the ruler.
17. Correct response: B
(Convert or estimate conversions of measures) 20. Correct response: C
Since 1 kilometer is equal to 1,000 meters, (Estimate and find length, perimeter, and area)
you can convert meters to kilometers by moving The garden is a rectangle, so its perimeter is
the decimal point three spaces to the left: twice the length plus twice the width. (2  10 ft)
2,564.0 m  2.564 km.  (2  16 ft)  20 ft  32 ft  52 ft.

Incorrect choices: Incorrect choices:


A reflects the idea that 1 km  10,000 m. A is the result of adding 10 ft  16 ft.
C reflects the idea that 1 km  100 m. B is the result of multiplying (2  10 ft) and
D reflects the idea that 1 km  10 m. then adding 16 ft.
D is the area of the rectangle (10 ft  16 ft)
rather than the perimeter.

70 Standardized Test Tutor: Math, Grade 4 © Michael Priestley, Scholastic Teaching Resources
21. Correct response: D 24. Correct response: A
(Estimate and find length, perimeter, and area) (Determine congruence and similarity)
The area of the fabric can be determined by In similar figures, the ratio of the sides will be
multiplying its length by its width: 8 ft  6 ft  equal. A triangle with sides that have a ratio of __ ​ 24 ​
48 sq ft. is similar to a triangle with sides that have a ratio
​ 48 ​because __
of __ ​ 24 ​= __
​ 48 ​.
Incorrect choices:
A is the result of adding the length and the Incorrect choices:
width: 8 ft  6 ft. B The triangle’s sides have a ratio of ​ __46 ​; __
​ 46 ​ __
​ 48 ​.
B is the result of using the formula for the ​ 28 ​; __
C The triangle’s sides have a ratio of __ ​ 28 ​ __
​ 48 ​.
area of a triangle instead of a rectangle:
​ 88 ​; __
D The triangle’s sides have a ratio of __ ​ 88 ​ __
​ 48 ​.
​ __12 ​(8 ft  6 ft).
C is the perimeter of the rectangle rather than
25. Correct response: B
the area: (2  8 ft)  (2  6 ft).
(Identify, classify, and describe solid figures and
their attributes)
22. Correct response: A
A rectangular prism has six faces, which can
(Identify and classify acute, obtuse, and right
be demonstrated with any rectangular box, such
angles and triangles)
as a cereal box.
The small panes of glass are triangles that
have one right angle, so they are right triangles. Incorrect choices:
Incorrect choices: A, C, and D represent misconceptions about
rectangular prisms, or a misunderstanding of
In B, an “acute” triangle has three angles less
the term faces.
than 90 degrees.
In C, an “obtuse” triangle has one angle 26. Correct response: C
greater than 90 degrees. (Locate and name points on a coordinate plane
In D, an “equilateral” triangle has three equal using ordered pairs)
sides and three equal angles. In an ordered pair, the first number (x) indi-
cates the number of units across and the second
23. Correct response: D number (y) indicates the number of units up or
(Identify, classify, and describe plane figures and down. On the grid, point C is 5 units across and
their attributes) 1 unit up.
The shape of the rear window is a polygon
with four sides, and only two of the sides are Incorrect choices:
parallel, so the figure must be a trapezoid. A is the result of following the coordinates in
the wrong order; point A is located at (1, 5),
Incorrect choices: not (5, 1).
A A hexagon has six sides. B Point B is located at (5, 5).
B A rectangle has two sets of equal sides D Point D is located at (1, 1).
and four right angles.
C A pentagon has five sides.

Standardized Test Tutor: Math, Grade 4 © Michael Priestley, Scholastic Teaching Resources
71
27. Correct response: C 30. Correct response: C
(Locate and name points on a coordinate plane (Solve problems involving length, perimeter,
using ordered pairs) area, time, temperature, volume, and
In an ordered pair, the first number (x) weight/mass)
indicates the number of units across and the To find the volume of a rectangular prism,
second number (y) indicates the number of multiply the length by the width by the height:
units up or down. In choice C, the peak of the 3 ft  2 ft  1.5 ft  9 ft3.
house is located at a point that is 7 units across
and 8 units up. Incorrect choices:
A is the result of multiplying 3 ft  1.5 ft.
Incorrect choices: B is the result of adding 3 ft  2 ft  1.5 ft.
A is the result of following the coordinates in D is the result of multiplying 3 ft by the height
the wrong order. The peak of the house is and 3 ft by the width:
located at (8, 7), not (7, 8). (3 ft  1.5 ft)  (3 ft  2 ft).
In B, the peak is located at (8, 8).
In D, the peak is located at (7, 7). 31. Correct response: A
(Solve problems involving length, perimeter,
area, time, temperature, volume, and
28. Correct response: D
weight/mass)
(Identify parallel, intersecting, and
Since there are 60 minutes in one hour, the
perpendicular lines)
number of minutes between 7:44 and 8:00 is
Parallel lines are lines that never intersect.
60  44  16 minutes. The number of minutes
On the map, the two streets that are parallel to
between 8:00 and 8:02 is 2  0  2 minutes. So
each other are Green Street and Apple Street.
the number of minutes between 7:44 and 8:02
Incorrect choices: is 16  2  18 minutes.
In A
 , Green Street and West Street are perpen- Incorrect choices:
dicular to each other. B represents an error in subtraction.
In B
 , Pine Street and Apple Street are two C is the result of subtracting 44  2,
streets that intersect. ignoring the hours.
In C
 , Pine Street and West Street are two D is the result of subtracting the times as if
streets that intersect. they were whole numbers: 802  744 
58 minutes.
29. Correct response: B
(Identify lines of symmetry) 32. Correct response: C
The glasses have one line of symmetry, a (Transform figures in the coordinate plane,
vertical line that passes through the middle of e.g., translation, rotation, reflection)
the bridge between the lenses. The placement A figure may turn, slide, or flip. When a
and shape of the bridge prevents the glasses figure is reflected, it “flips” over a line. The figure
from having any other lines of symmetry. becomes a “mirror” image and appears to be the
opposite of the original figure.
Incorrect choices:
A, C, and D represent misconceptions about Incorrect choices:
lines of symmetry or a misunderstanding In A, the figure has been rotated or turned 90,
of what symmetry means. not reflected.
In B, the figure has been translated or slid,
not reflected.
In D, the figure has been rotated or turned
270, not reflected.
72 Standardized Test Tutor: Math, Grade 4 © Michael Priestley, Scholastic Teaching Resources
33. Correct response: D 35. (continued)
(Match 3-dimensional objects and their Incorrect choices:
2-dimensional representations) A Angelo saw more cardinals and chickadees
The figure in answer choice D can be folded than blue jays.
to form a rectangular prism like the one shown.
B There were less than half as many
This may be demonstrated by cutting this figure
chickadees as sparrows.
from a piece of paper and folding it.
D More chickadees than blue jays visited
Incorrect choices: the feeder.
In A, this figure cannot be folded to form
a rectangular prism (it would have only 36. Correct response: D
three faces). (List all possible outcomes or construct sample
spaces using lists, charts, frequency tables, and
In B, this figure cannot be folded to form a
tree diagrams)
rectangular prism (both “flaps” would fold
To find all of the possible outcomes from
down over the same end).
a tree diagram, follow every path from the top
In C, this figure cannot be folded to form to the bottom. There are 8 paths in this tree
a rectangular prism (it has sides that diagram, and 8 possible outcomes.
are not perpendicular or do not have
right angles). Incorrect choices:
A represents the list of outcomes in the final
34. Correct response: D stage of the tree diagram listed in pairs.
(Interpret data presented in line, bar, and circle
B represents the outcomes in each stage of
graphs, pictographs, tables, charts)
the tree diagram in a group.
There are 6 pictures of popcorn next to
Wednesday in the pictograph, and each picture C lists only 4 of the 8 possible outcomes.
represents 20 tubs of popcorn, so there were 6 
20  120 tubs of popcorn sold on Wednesday. 37. Correct response: B
(Find probabilities)
Incorrect choices: There are 23 socks in the drawer, and there
A is the result of counting the number of are more green socks than socks of any other
pictures of popcorn next to Wednesday color. The probability of picking a yellow sock
is __ 3  ​, a green sock is __
​ 23 9  ​, a blue sock is __
​ 23 6  ​, and
​ 23
and not multiplying by 20.
a white sock is ​ __ 5  ​. Danny is most likely to pick
B is the total number of pictures of popcorn 23
in the pictograph. a green sock.
C would be correct if each picture of popcorn Incorrect choices:
represented 10 tubs instead of 20.
A The probability of picking a yellow sock
3  ​, which is less likely than __
is ​ __ 9  ​.
​ 23
35. Correct response: C 23
(Interpret data presented in line, bar, and circle C The probability of picking a blue sock
is __ 6  ​, which is less likely than __
​ 23 9  ​.
​ 23
graphs, pictographs, tables, charts)
The section of the circle graph labeled D The probability of picking a white sock
“Sparrows” takes up more than half of the is __ 5  ​, which is less likely than __
​ 23 9  ​.
​ 23
graph, so there were more sparrows at
Angelo’s birdfeeder than all the other kinds
of birds combined.

Standardized Test Tutor: Math, Grade 4 © Michael Priestley, Scholastic Teaching Resources
73
38. Correct response: D 41. Correct response: C
(Determine and describe the mean, median, (Identify, describe, and extend numerical and
mode, and range of data) geometric patterns)
To find the mean of the times on the The pattern is a sequence of polygons that
table, add all of the times and divide the sum goes in the following order: rectangle, triangle,
(80 seconds) by the number of times (5): trapezoid. Since the last figure shown in the pat-
80 seconds  5  16 seconds. tern is a triangle, the next must be a trapezoid.
The second time that the rectangle and triangle
Incorrect choices: appear in the pattern, they are rotated 90 to the
A represents the range of the data (22  12). right from their original positions. Therefore,
the next figure in the pattern will be a trapezoid
B is the mode of the data (12 appears twice in
rotated 90 to the right from its original position.
the data).
C is the median of the data. Incorrect choices:
A is a trapezoid that has not been rotated; it
39. Correct response: B will appear as the 9th figure in the pattern.
(Determine and describe the mean, median,
B is a rectangle that has not been rotated; it
mode, and range of data)
will appear as the 7th figure in the pattern.
The range of the prices is the difference
between the highest price ($2.10) and the lowest D is a triangle that has been rotated 180.
price ($0.70): $2.10  $0.70  $1.40.
42. Correct response: A
Incorrect choices: (Apply basic properties and order of operations
A represents the difference between the first with algebraic expressions)
and last prices listed ($1.89  $1.59). By the commutative property, 30  3x  120
is equal to 3x  30  120.
C is the last price listed on the chart.
D is the highest price listed on the chart. Incorrect choices:
In B, the number sentence is not equivalent;
40. Correct response: A 3(30  x) is equivalent to 90  3x.
(Collect, organize, display, and interpret data
In C, the number sentence is not equivalent;
to solve problems)
3x(30) is equivalent to 90x.
The circle graph in choice A shows the
correct percentages for each hair color. In D, the number sentence would be
equivalent to 30  3x  120.
Incorrect choices:
In B
 , the graph shows incorrect percentages; 43. Correct response: C
for example, “black hair” is shown as 50% (Use algebraic expressions, patterns, and simple
instead of 45%. equations and inequalities to represent problem
situations)
In C
 , the graph shows incorrect percentages;
The total number of cards Miwa buys is the
for example, “black hair” is shown as about
number of packs (p) by the number of cards
40% instead of 45%.
in each pack (8), or 8p. The number of cards
In D
 , the graph shows incorrect percentages; she will buy must be at least 100, so 8p must be
for example, “black hair” is shown as 55% greater than or equal to 100, which can be writ-
instead of 45%. ten as 8p  100.

74 Standardized Test Tutor: Math, Grade 4 © Michael Priestley, Scholastic Teaching Resources
43. (continued) 45. Correct response: B
Incorrect choices: (Evaluate the reasonableness of a solution)
A has the wrong inequality sign. To find the most reasonable estimate, you
can round the number of rows to 120 and the
B does not represent the problem situation;
number of seats per row to 10: 120 rows 
it adds the number of baseball cards (100)
10 seats  1,200 seats.
and the number of cards per pack (8).
D does not represent the problem situation; Incorrect choices:
it adds the number of cards per pack (8) A reflects an error in multiplication or place
and the number of packs (p) instead of value, resulting in only one seat per row.
multiplying.
In C, the estimate is too high; it reflects an
error in multiplication or place value,
44. Correct response: D based on 100 seats per row.
(Represent and describe mathematical
relationships with lists, tables, charts, graphs, In D, the estimate is too high; it reflects an
and diagrams) error in multiplication or place value,
The amount of money Darrell’s parents based on 1,200 rows or 1,000 seats per row.
will give to charity is $5 by the number of miles
he walks. If the input (miles) in the table is n,
then the output (dollars) will be 5n. In other
words, the amount in dollars should be 5
multiplied by the number of miles. The table
in choice D shows this relationship: 1  5  5;
2  5  10; and 4  5  20.

Incorrect choices:
In A, the table shows the relationship if
Darrell’s parents gave $5 for every 2 mile
she walked.
In B, the numbers in the first column are
correct, but the numbers in the next two
columns show the relationship n  5.
In C, the table shows the relationship n  5
(adding $5 to the number of miles).

Standardized Test Tutor: Math, Grade 4 © Michael Priestley, Scholastic Teaching Resources
75
Test 2 Answer Key

1. B 10. C 19. A 28. D 37. A

2. D 11. D 20. D 29. B 38. B

3. B 12. C 21. D 30. D 39. C

4. B 13. C 22. A 31. C 40. B

5. C 14. B 23. C 32. A 41. D

6. A 15. C 24. B 33. C 42. A

7. B 16. C 25. A 34. B 43. D

8. D 17. B 26. C 35. D 44. B

9. A 18. A 27. A 36. D 45. C

Answer Key Explanations


1. Correct response: B 2. (continued)
(Read and write whole numbers and decimals) Incorrect choices:
The standard form of three hundred eighty- A The seventh hash mark on the number line
four thousand four hundred has a 3 in the represents ​ __78 ​.
hundred thousands place (300,000), an 8 in
B The sixth hash mark on the number line
the ten thousands place (80,000), a 4 in the
​ 68 ​, or __
represents __ ​ 34 ​.
thousands place (4,000), and a 4 in the hundreds
place (400): 384,400. C The fourth hash mark on the number line
​ 48 ​, or __
represents __ ​ 12 ​.
Incorrect choices:
A is three thousand eight hundred forty-four. 3. Correct response: B
(Compare and order whole numbers, fractions,
C is three hundred eighty-four million four
and decimals)
hundred thousand.
Group 2’s bean plant was __ ​ 94 ​inches tall, which
D is three hundred eighty-four million 1
__
is equal to 2 ​ 4 ​, or 2.25. This number is greater
four hundred. than the height of the other bean plants (2 in.,
0.8 in., and __3  ​in.).
​ 10
2. Correct response: D
(Compare and order whole numbers, fractions, Incorrect choices:
and decimals) A Group 1’s bean plant is 2 inches tall, which
On this number line, the space between is less than ​ __94 ​.
0 and 1 is divided into eight units, or eighths. 8  ​, inch tall,
C Group 3’s bean plant is 0.8, or __
​ 10
​ 58 ​.
Point R represents 5 units out of 8, or __
​ 94 ​.
which is less than __
D Group 4’s bean plant is __ 3  ​, or 0.3, inch tall,
​ 10
​ 94 ​.
which is less than __

76 Standardized Test Tutor: Math, Grade 4 © Michael Priestley, Scholastic Teaching Resources
4. Correct response: B 6. (continued)
(Represent, convert, and find equivalent fractions Incorrect choices:
and mixed numbers) B is incorrect because 68 is not a multiple
To convert an improper fraction to a of 8.
mixed number, divide the numerator by the
C is incorrect because 76 is not a multiple
denominator: 35  8  4 with 3 left over. 4
of 8.
is the whole number, and the remainder of
​ 38 ​, so the mixed number is 4 __
3 represents __ ​ 38 ​. D is incorrect because 92 is not a multiple
of 8.
Incorrect choices:
A is the result of rearranging the digits in the 7. Correct response: B
improper fraction. (Add, subtract, multiply, and divide
whole numbers)
C is the result of dividing 35 by 8 and
To find the number of tickets sold at the
then putting the remainder (3) over the
theater for one movie, divide the total amount
numerator of the improper fraction (35)
of ticket sales by the price of each ticket:
instead of over the denominator (8).
$3,276  9  364.
D reflects an error in division and putting
the remainder (5) over the numerator of Incorrect responses:
the improper fraction (35) instead of the
A reflects an error in long division.
denominator (8).
C is the result of adding 3,276  9 instead
of dividing.
5. Correct response: C
(Identify and use place value) D is the result of multiplying 3,276 by 9.
This number can be written in expanded
form as 6,000,000  500,000  20,000  1,000  8. Correct response: D
900  90  8. The 5 is in the hundred thousands (Add and subtract fractions and mixed numbers)
place. To add the fractions, first give them common
denominators. The least common multiple of 5
Incorrect choices: 3  ​ __ 3  ​. Then add the
​ 25 ​ __
and 10 is 10: __ 4  ​and __
​ 10 ​ 10 ​ 10
A The 1 is in the thousands place. 4  ​ __3  ​ __ 7  ​.
numerators: ​ __
10
​ 10 ​ 10
B The 2 is in the ten thousands place.
D The 9 is in the hundreds or the tens place. Incorrect choices:
A is the result of adding the numerators and
6. Correct response: A the denominators (2  3 over 5  10).
(Identify prime numbers, factors, and multiples) B is the result of converting the fraction ​ __ 3  ​
The total number of juice boxes 10
improperly to ​ __15 ​(​ __25 ​ __ ​ 35 ​).
​ 15 ​ __
Mr. Chowdhury bought must be a multiple
C is the result of multiplying the fractions.
of 8 (in other words, the number must be evenly
divisible by 8). 56 is a multiple of 8 (7  8  56),
9. Correct response: A
so he could have bought 7 packages of 8 juice
(Add, subtract, multiply, and divide decimals)
boxes.
To find the difference in the weights of the
backpacks, subtract the weight of Bart’s back-
pack from the weight of Kerby’s backpack,
making sure to line up the decimal points:
3.56 kg  2.6 kg  0.96 kg.

Standardized Test Tutor: Math, Grade 4 © Michael Priestley, Scholastic Teaching Resources
77
9. (continued) 12. (continued)
Incorrect choices: Incorrect choices:
B is the result of subtracting 3.56  2.06. A Adding the two numbers does not give the
C is the result of forgetting to change the same result as subtracting them.
3 to a 2 after borrowing 1 to subtract 0.6 B reflects a misunderstanding of the
from 0.5. inverse relationship between subtraction
D is the result of subtracting 3.56  0.26. and addition.
D Multiplication and subtraction do not have
10. Correct response: C an inverse relationship.
(Solve problems involving integers)
The difference in temperatures can be found 13. Correct response: C
by subtracting the temperature in the freezer (Solve multi-step problems involving whole
from the temperature in the refrigerator: numbers, fractions, decimals)
3C  (2C)  3C  2C  5C. The cost of the shirts before the coupon is
$24.80  4  $99.20. Subtract $5.95 from the to-
Incorrect choices:
tal to find the amount Lana spent after she used
A is the result of subtracting 3C  2C. her coupon: $99.20  $5.95  $93.25.
B is the temperature in the refrigerator.
Incorrect choices:
D is the result of adding the zero on a
thermometer as a degree (2, –1, 0, 1, 2, A is the result of subtracting the value of
and 3 appear to be 6 degrees). the coupon from the price of one shirt:
$24.80  $5.95.
11. Correct response: D B is the result of subtracting the value of
(Estimate using whole numbers and decimals) the coupon from the price of each shirt:
To estimate the total price, round each price ($24.80  $5.95)  4.
to the nearest tenth and then add them together.
D is the total cost of the shirts before
The apple for $0.98 can be rounded to $1.00; the
subtracting the value of the coupon.
granola bar for $1.39 can be rounded to $1.40,
and the drink for $1.52 can be rounded to $1.50:
14. Correct response: B
$1.00  $1.40  $1.50  $3.90.
(Solve multi-step problems involving whole
Incorrect choices: numbers, fractions, decimals)
The number of students on each team is
A is the result of rounding each price
the total number of students (288) divided by
to $1.00.
the number of teams (12): 288  12  24. The
B is the result of inconsistent rounding and number of students that will play in the kickball
then adding $1.00  $1.00  $1.50. tournament from each team is 24  __ ​ 23 ​ 16.
C is the result of inconsistent rounding and
then adding $0.90  $1.30  $1.50. Incorrect choices:
A is the result of multiplying the number
12. Correct response: C of teams (12)  ​ __23 ​.
(Apply the properties of operations) ​ 23 ​.
C reflects an error in multiplying 24  __
Addition and subtraction have an inverse
D is the number of students on each team
relationship, so 50  23  p is equivalent to
before multiplying by ​ __23 ​.
p  23  50.

78 Standardized Test Tutor: Math, Grade 4 © Michael Priestley, Scholastic Teaching Resources
15. Correct response: C 18. Correct response: A
(Solve number sentences with one variable) (Select appropriate unit for measuring weight/
To solve the number sentence, subtract 7 mass, capacity, length, perimeter, and area)
from both sides of the equation: The area of a town can best be expressed
by the largest unit of measurement given:
7  m ( 7)  21 ( 7) square kilometers.
m  14
Incorrect choices:
Incorrect choices: B a nd C are incorrect because these units are
A is the result of dividing 21  7. too small to be practical; meters and feet
are more appropriate for measuring the
B is a number in the equation.
area of a field or a house lot.
D is the result of adding 21  7.
D A millimeter is considerably smaller than
an inch.
16. Correct response: C
(Tell time and find elapsed time)
19. Correct response: A
The clock at Time 1 reads 8:25, and the
(Use rulers and other instruments to measure
clock at Time 2 reads 9:30. To find the difference
accurately)
between the times, subtract 8:25 from 9:30. The
Each hash mark on the scale represents
race lasted 1 hour 5 minutes.
0.1 lb. The pointer is pointing to the seventh
hash mark after the mark that denotes 2 lb.
Incorrect choices:
The weight of the potatoes is 2.7 lb.
A is the result of subtracting the minutes
and disregarding the hours. Incorrect choices:
B is the result of subtracting the minutes B is the result of thinking that the arrow on
and counting them as hours. the scale points to the eighth hash mark
D is the result of finding the difference to be instead of the seventh.
1  ​of an hour, then converting __
1 hour and ​ __ 1  ​
​ 12
12 C is the result of reading the scale backward
to 12 minutes instead of 5. from 3 and thinking that the arrow points
to the second hash mark instead of the
17. Correct response: B third.
(Convert or estimate conversions of measures) D is the result of reading the scale backward
One meter is approximately equal to from 3.
one yard, or 3 feet. If the house is 30 feet tall,
then its approximate height in meters is
20. Correct response: D
30  3  10 meters.
(Estimate and find length, perimeter, and area)
The wall is a rectangle, so its area can be
Incorrect choices:
determined by multiplying its length by its
A is the result of assuming that one meter height: 8 ft  13 ft  104 sq ft.
equals 10 feet and dividing 30 ÷ 10.
C is the result of assuming that one meter is Incorrect choices:
about the same as one foot. A is the result of adding 8 ft  13 ft instead
D is the result of multiplying 30 ft  3 instead of multiplying.
of dividing. B is the perimeter of the wall (2  8 ft plus
2  13 ft) instead of the area.
C is the result of using the formula for area
of a triangle: ​ __12 ​(8 ft  13 ft).

Standardized Test Tutor: Math, Grade 4 © Michael Priestley, Scholastic Teaching Resources
79
21. Correct response: D 24. Correct response: B
(Estimate and find length, perimeter, and area) (Determine congruence and similarity)
The length of the field can be rounded to Congruent figures are the same shape and
70 meters, and the width can be rounded to the same size. The figure in answer choice B has
60 meters. The perimeter is 2 times the length the same dimensions as the figure above.
plus 2 times the width, so a good estimate of the
perimeter is 2(70 m)  2(60 m)  260 m. Travis Incorrect choices:
ran the perimeter of the field, so 260 m is a good In A, the figure is similar to the figure
estimate of how far he ran. above but not congruent.
In C and D, the figures have different
Incorrect choices:
dimensions from the figure above
A is the result of rounding both numbers and thus cannot be congruent.
down and adding 70 m  50 m.
B is the result of rounding both numbers to 25. Correct response: A
the nearest ten and adding the length plus (Identify, classify, and describe solid figures and
width (70 m  60 m). their attributes)
C is the result of rounding 58 ft down to 50 The basketball is a sphere; a cross section of
feet and multiplying 2(70 m)  2(50 m). a sphere is always a circle.

Incorrect choices:
22. Correct response: A
(Identify and classify acute, obtuse, and right B This figure is a square pyramid.
angles and triangles) C This figure is a cone.
An obtuse angle is an angle that is greater
D This figure is a cylinder.
than 90. Angle M is greater than 90.

Incorrect choices: 26. Correct response: C


(Locate and name points on a coordinate plane
B Angle N is a right angle.
using ordered pairs)
C Angle O is a right angle. In an ordered pair, the first number (x)
D Angle P is an acute angle. indicates the number of units across, and the
second number (y) indicates the number of
23. Correct response: C units up or down. On the grid, point R is
(Identify, classify, and describe plane figures and located at (4, 2).
their attributes)
Incorrect choices:
A parallelogram is a polygon with two pairs
of parallel sides. The polygon in answer choice A is the result of reading the y coordinate first
C is a trapezoid and does not have two sets of (up 2) and then the x coordinate (across 4).
parallel sides. B is the result of reading only the
x coordinate (4).
Incorrect choices:
D is the result of misreading the coordinates
In A
 , B, and D, the figures are all polygons as 4 across and 4 up.
with two sets of parallel sides and thus
are parallelograms.

80 Standardized Test Tutor: Math, Grade 4 © Michael Priestley, Scholastic Teaching Resources
27. Correct response: A 30. (continued)
(Locate and name points on a coordinate plane Incorrect choices:
using ordered pairs) A shows the figure translated one unit to the
In an ordered pair, the first number (x) right and one unit down.
indicates the number of units across, and the
B shows the figure translated three units to
second number (y) indicates the number of
the right and one unit down.
units up or down. On the grid, vertex Q has an
x-coordinate of 2 and a y-coordinate of 5. C shows the figure translated two units to
the right.
Incorrect choices:
B Vertex R is located at (5, 2). 31. Correct response: C
(Solve problems involving length, perimeter,
C Vertex S is located at (5, 0).
area, time, temperature, volume, and
D Vertex T is located at (2, 0). weight/mass)
The area of the lawn to be mowed is the
28. Correct response: D area of the whole property minus the area taken
(Identify parallel, intersecting, and up by the house. The area of the property is
perpendicular lines) 10 m  14 m  140 m2. The area of the house
Perpendicular lines intersect at a 90-degree is 8 m  8 m  64 m2. 140 m2  64 m2  76 m2.
angle. The only one of these letters that has
perpendicular lines is H. Incorrect choices:

Incorrect choices: A is the result of subtracting the length


and width of the house (8  8) from the
A The letter W has lines that intersect at length and width of the lawn (10  14):
acute angles. 24  16  8.
B The letter X has lines that intersect to B is the difference between the perimeter of
form a pair of acute angles and a pair the lawn and the perimeter of the house.
of obtuse angles.
D is the result of adding the two areas instead
C The letter N contains parallel lines and of subtracting: 140 m2  64 m2.
lines that intersect at acute angles.
32. Correct response: A
29. Correct response: B (Solve problems involving length, perimeter,
(Identify lines of symmetry) area, time, temperature, volume, and
The figure shown is a trapezoid, and it has weight/mass)
only one line of symmetry (a vertical line drawn The amount that the jam must increase in
through the center of the figure). temperature is the temperature it needs to reach
Incorrect choices: (218F) minus its current temperature (199.3):
218F  199.3F  18.7F.
A, C, and D reflect misconceptions about lines
of symmetry. Incorrect choices:
B reflects an error in subtraction.
30. Correct response: D
(Transform figures in a coordinate plane, C is the result of subtracting 199.3F  21.8F.
e.g., translation, rotation, reflection) D is the result of adding 199.3F  218F
The figure in answer choice D has been instead of subtracting.
translated two units to the right and one
unit down.

Standardized Test Tutor: Math, Grade 4 © Michael Priestley, Scholastic Teaching Resources
81
33. Correct response: C 35. Correct response: D
(Match 3-dimensional objects and their (Interpret data presented in line, bar, and circle
2-dimensional representations) graphs, pictographs, tables, charts)
The figure in answer choice C can be folded Betsy’s handstand time was 5 seconds in
to form a triangular pyramid like the one shown. both June and September.
This may be demonstrated by cutting this figure
from a piece of paper and folding it. Incorrect choices:
A Betsy’s times did not increase steadily; her
Incorrect choices: time even fell from August to September.
In A
 , this figure cannot be folded to form B The greatest increase in Betsy’s times was
a triangular pyramid (it would have only between September and October.
three faces).
C Betsy’s time was greatest in October,
In B
 , this figure cannot be folded to form not August.
a triangular pyramid (the triangular “base”
is not drawn in the correct location).
36. Correct response: D
In D
 , this figure cannot be folded to form (List all possible outcomes or construct sample
a triangular pyramid (it would have spaces using lists, charts, frequency tables, and
five faces). tree diagrams)
Each time the arrow is spun, it could land on
34. Correct response: B any of the 3 sections. If the arrow is spun twice,
(Interpret data presented in line, bar, and circle there are 3  3  9 possible outcomes, as listed
graphs, pictographs, tables, charts) in answer choice D.
The cost for a child visiting the main aquari-
um, sea otter exhibit, and dolphin show is $8  Incorrect choices:
$3  $4  $15; this is the same price as an adult A lists all possible combinations of the letters.
ticket to the main aquarium.
B lists all the possible outcomes if repetition
Incorrect choices: were not possible.
A It costs only $5 for a senior ticket to the C lists all the possible outcomes for three
dolphin show, which is less than the spins if repetition were not possible.
$8 child’s ticket to the main aquarium.
37. Correct response: A
C It costs $10 for a senior to visit the main (Find probabilities)
aquarium, which is less than the total of There are a total of 10 candies in the bag, and
$12 it would cost for an adult to visit the 3 of the 10 candies are blue. The probability of
sea otter exhibit and the dolphin show. picking a blue candy is 3 out of 10, or __3  ​.
​ 10
D It would cost $9 for a senior to visit the sea
otter exhibit and the dolphin show, which Incorrect choices:
is not the same as the $8 ticket for a child B represents the number of blue candies over
to visit the main aquarium. the number of red and yellow candies.
C represents the probability of choosing a red
5  ​, or __
candy (​ __ ​ 12 ​).
10
D represents the probability of choosing
a candy that is not blue.

82 Standardized Test Tutor: Math, Grade 4 © Michael Priestley, Scholastic Teaching Resources
38. Correct response: B 41. Correct response: D
(Determine and describe the mean, median, (Identify, describe, and extend numerical and
mode, and range of data) geometric patterns)
To find the mean, add all of the numbers In the number pattern, each number is 3
of pages (125  96  157  278) and divide the greater than the number before. To find the next
sum (656) by the number of books (4); 656  4 number in the pattern, add 3 to the last number
 164. listed. The correct expression is 12  3.

Incorrect choices: Incorrect choices:


A represents the median of the data. A is the result of multiplying the first number
in the pattern (3) by the total number of
C represents the range of the data.
numbers in the pattern (4).
D represents the total number of pages.
B is the result of multiplying 3 times the last
number instead of adding.
39. Correct response: C
(Determine and describe the mean, median, C represents a misunderstanding of the
mode, and range of data) number pattern or how to describe it.
The mode is the number that occurs the
greatest number of times in a data set. The 42. Correct response: A
number 9 appears 4 times in this data set, (Apply basic properties and order of operations
more than any other number. with algebraic expressions)
By the distributive property, $0.50(b  d)
Incorrect choices:  $10.00 is equivalent to $0.50b  $0.50d 
A The number 8 appears 3 times in the $10.00.
data set.
Incorrect choices:
B 8.5 is the median of the data.
In B, the number sentence is not equivalent
D 11 is the range of the data.
because (b  d) is not the same as (bd).
In C
 and D, the number sentences are not
40. Correct response: B
equivalent; they represent misunderstand-
(Collect, organize, display, and interpret data
ings of the basic properties of operations.
to solve problems)
The bar graph shows that 7 students chose
a hamster and 3 students chose a fish. The 43. Correct response: D
difference between them is 7  3  4. (Use algebraic expressions, patterns, and
simple equations and inequalities to represent
Incorrect choices: problem situations)
A is the number of students who chose a fish. The number of stickers each student will
receive is the total number of stickers divided by
C is the difference between the number
the number of students, or 360  s.
of students who chose a lizard and the
number of students who chose a fish Incorrect choices:
(9  3  6).
A reflects adding the number of stickers and
D is the number of students who number of students instead of dividing.
chose a hamster.
B reflects subtracting the number of
students from the number of stickers
instead of dividing.
C reflects multiplying the number of stickers
by number of students instead of dividing.

Standardized Test Tutor: Math, Grade 4 © Michael Priestley, Scholastic Teaching Resources
83
44. Correct response: B
(Represent and describe mathematical
relationships with lists, tables, charts, graphs,
and diagrams)
Answer choice B shows the relationship
between the number of hours and number of
miles walked. Every y-value on the line is 2 times
its corresponding x-value. For example, if he
walks 3 hours, he goes 6 miles; in 4 hours, he
walks 8 miles.

Incorrect choices:
A shows that the number of miles he walks is
2 miles, regardless of the number of hours.
C shows that the number of hours walked is
2 hours, regardless of the number of miles.
D shows the opposite relationship
(for every 2 hours he walks, he goes 1 mile).

45. Correct response: C


(Evaluate the reasonableness of a solution)
The area of the swimming pool is its length
times its width. If the length is greater than the
width, then the area must be at least 60 ft  60
ft  3,600 sq ft. But the length cannot be greater
than 100 ft, so the area can not be greater than
6,000 sq ft (60 ft  100 ft). The best estimate of
the area is 4,800 sq ft, which is between 3,600
and 6,000 sq ft.

Incorrect choices:
A is the result of adding the width and the
greatest possible length (60  100).
B is the perimeter (based on 60 ft  100 ft)
instead of the area.
D is an overestimate because the dimensions
cannot exceed 60 ft  100 ft.

84 Standardized Test Tutor: Math, Grade 4 © Michael Priestley, Scholastic Teaching Resources
Test 3 Answer Key

1. C 10. B 19. B 28. C 37. B

2. C 11. C 20. A 29. D 38. A

3. A 12. B 21. D 30. B 39. C

4. D 13. B 22. C 31. A 40. D

5. C 14. D 23. A 32. C 41. B

6. B 15. D 24. C 33. C 42. D

7. D 16. A 25. D 34. A 43. A

8. A 17. A 26. B 35. B 44. C

9. D 18. B 27. A 36. C 45. B

Answer Key Explanations


1. Correct response: C 2. (continued)
(Read and write whole numbers (to the millions) Incorrect responses:
and decimals) A 1.8 would be located to the left of 2.0.
The standard form of five hundred sixty
B 2.2 would be located closer to 2.0 than 2.5.
thousand three hundred twenty will have a 5
in the hundred thousands place (500,000), a 6 D 2.8 would be located to the right of 2.5 and
in the ten thousands place (60,000), a 0 in the closer to 3.0.
thousands place, a 3 in the hundreds place
(300), a 2 in the tens place (20), and a 0 in the 3. Correct response: A
ones place: 560,320. (Compare and order whole numbers, fractions,
and decimals)
Incorrect choices: All of these fractions can be compared by
A is five thousand six hundred thirty-two. putting them over the common denominator
of 90: __ 3  ​ __
​ 10 ​ 27  ​; __ ​ 72
​ 4 ​ __ ​ 60
​ 2 ​ __
 ​; __ ​ 59 ​ __
 ​; and __ ​ 50  ​. Sasha
B is fifty-six thousand three hundred twenty. 90 5 90 3 90 90
D is five hundred sixty-three thousand ​ 27
read the least because __
90
 ​, or __ 3  ​, is the smallest
​ 10
twenty. fraction.

Incorrect choices:
2. Correct response: C
(Compare and order whole numbers, fractions, B José read ​ __45 ​of the book, which is greater
than __ 3  ​.
​ 10
and decimals)
On this number line, each hash mark repre- C Fran read __ ​ 23 ​of the book, which is greater
than __ 3  ​.
​ 10
sents 0.5. Point H is between 2.0 and 2.5 on the
number line, but closer to 2.5. The number 2.4 is D Neal read __ ​ 59 ​of the book, which is greater
between 2.0 and 2.5, closer to 2.5. than __ 3  ​.
​ 10

Standardized Test Tutor: Math, Grade 4 © Michael Priestley, Scholastic Teaching Resources
85
4. Correct response: D 7. Correct response: D
(Represent, convert, and find equivalent fractions (Add, subtract, multiply, and divide
and mixed numbers) whole numbers)
Equivalent fractions can be reduced to the To find the number of seats on the train,
same fraction. __ 8  ​can be reduced to __
​ 12 ​ 23 ​(dividing multiply the number of cars (7) by the number
​ 24
the numerator and denominator by 4), and __
36
 ​ of seats on each car (128): 7  128  896.
can be reduced to ​ __23 ​(dividing the numerator and
Incorrect responses:
denominator by 12).
A i s the result of adding 7  128 instead
Incorrect choices: of multiplying.
A is incorrect because ​ __34 ​cannot be reduced B reflects an error in multiplication
8  ​. (not carrying the 5 from 7  8  56
to ​ __23 ​and is not equivalent to __
​ 12
8  ​are not
or the 1 from 7  2  14).
B is incorrect because __ ​ 12
8
  ​and __
​ 12
8  ​ C r eflects an error in multiplication
equivalent. __​ 12  ​can be simplified to 1 __ ​ 12 ​; __
​ 12
8 (not carrying 1 from 7  2  14).
can be reduced to __ ​ 23 ​.
​ 49 ​cannot be reduced
C is incorrect because __ 8. Correct response: A
and is not equivalent to __ 8  ​.
​ 12 (Add and subtract fractions and mixed numbers)
To subtract the fractions, first give them
5. Correct response: C common denominators. The least common
(Identify and use place value) ​ 13 ​ __
multiple of 12 and 3 is 12: __ 4  ​and __
​ 12 5  ​ __
​ 12 5  ​.
​ 12
The tenths place is one digit to the right of 5  ​ __4  ​ __ 1  ​.
Then subtract the numerators: __
​ 12 ​ 12 ​ 12
the decimal. In $1,653.89, the number that is
one digit to the right of the decimal is 8. That Incorrect choices:
represents 0.8, or 8 tenths, or __8  ​.
​ 10
B is the result of converting ​ __13 ​incorrectly to __ 3  ​
​ 12
and subtracting __ 5  ​ __
​ 12 ​  3  ​ __ 2  ​, or __
​ 12 ​ 16 ​.
Incorrect choices:
12
A The 3 is in the ones place. C is the result of converting ​ __13 ​incorrectly to __ 1  ​
​ 12
and subtracting __ 5  ​ __
​ 12 1  ​ __
​ 12 4  ​, or __
​ 12 ​ 13 ​.
B The 5 is in the tens place.
D The 9 is in the hundredths place. D is the result of subtracting the numerators
(5  1) and denominators (12  3).
6. Correct response: B
(Identify prime numbers, factors, and multiples) 9. Correct response: D
The number of students in Grade 2 (23) is a (Add, subtract, multiply, and divide decimals)
prime number because it is divisible only by 1 To find the total cost, add all of the prices
and itself. together, being careful to line up the decimals:
$25.95  $7.39  $12.60  $45.94.
Incorrect choices:
Incorrect choices:
A The number of students in Grade 1 (27) is
divisible by 3 and 9. A is the result of adding only two prices:
$25.95  $7.39.
C The number of students in Grade 3 (34) is
divisible by 2 and 17. B is the result of adding $25.95  $7.39 
$1.26.
D The number of students in Grade 4 (33) is
divisible by 3 and 11. C is the result of an error in addition
(tens are not carried over).

86 Standardized Test Tutor: Math, Grade 4 © Michael Priestley, Scholastic Teaching Resources
10. Correct response: B 13. Correct response: B
(Solve problems involving integers) (Solve multi-step problems involving whole
The number of spaces Nicole should move numbers, fractions, decimals)
can be determined by finding the sum of the First, find the total number of balloons
two cards: (1)  4  3. Ozzie bought by multiplying the number of
packages (6) times the number of balloons
Incorrect choices: in each package (12): 6  12  72. Since Ozzie
A is the absolute value of the first card (1). gives the same number to each guest with none
left over, the number of balloons each guest
C is the number on the second card (4).
receives is the total number of balloons (72)
D is the result of subtracting 4  (1), or divided by the number of guests (9): 72  9  8.
4  1.
Incorrect choices:
11. Correct response: C A is the result of adding 6  12 (instead of
(Estimate using whole numbers and decimals) multiplying) and then dividing by 9.
To estimate the total weight, round each
C is the result of adding 6  12 and then
weight to the nearest ten and then add the
subtracting 9 (instead of dividing).
weights together. Ned’s weight of 98 pounds
can be rounded to 100 lb; his brother’s weight D is the result of adding 6  12 and then
of 111 pounds can be rounded to 110 lb, and his adding 9 (instead of dividing).
father’s weight of 169 pounds can be rounded to
170 lb. 100  110  170  380. 14. Correct response: D
(Solve multi-step problems involving whole
Incorrect choices: numbers, fractions, decimals)
A is the result of rounding each weight to 100. The total distance Cherry travels to school
is 1.8 km  5.6 km  7.4 km. Since she makes
B is the result of rounding the numbers down
the same trip on the way home, the total
and adding: 90  110  160.
distance she travels going to school and back
D is the result of rounding the numbers to the is 7.4 km  2  14.8 km.
nearest 100 and adding: 100  100  200.
Incorrect choices:
12. Correct response: B A reflects a mistake in adding 1.8 km and
(Apply the properties of operations) 5.6 km, and not multiplying by 2.
Because addition and subtraction have an
B represents the distance one way; it was not
inverse relationship, 24  m  45 is equivalent
multiplied by 2.
to 45  m  24.
C represents a mistake in adding 1.8 km and
Incorrect choices: 5.6 km (not carrying the 1 and getting 6.4)
A Subtracting m from 24 is not the same as and then multiplying by 2.
adding m  24.
C Adding m to 45 is not the same as adding
15. Correct response: D
(Solve number sentences with one variable)
m  24.
To solve the number sentence, add 6 to both
D reflects a misunderstanding of the sides of the equation:
inverse relationship between subtraction
and addition. n  6 ( 6)  18 ( 6)

n  24

Standardized Test Tutor: Math, Grade 4 © Michael Priestley, Scholastic Teaching Resources
87
15. (continued) 18. (continued)
Incorrect choices: Incorrect choices:
A is the result of dividing 18  6. A A meter is a little longer than one yard—too
B is a number in the equation. large to measure a shoe.
C is the result of subtracting 18  6. C A centimeter equals 10 millimeters,
so millimeters are too small to measure
a shoe.
16. Correct response: A
D A kilometer is about ​ __6  ​of a mile—too large
(Tell time and find elapsed time) 10
Since there are 60 minutes in 1 hour, to measure a shoe.
the time between 4:45 and 6:02 is 1 hour
17 minutes. (There is 1 hour between 4:45 is 19. Correct response: B
5:45; an additional 15 minutes brings the (Use rulers and other instruments to measure
time to 6:00 (45  15  60), and an additional accurately)
2 minutes brings the time to 6:02. 15 minutes  The carrot is lined up with the 1-inch hash
2 minutes  17 minutes.) mark on one end and the 5.5-inch hash mark
on the other end. Its length is 5.5 inches 
Incorrect choices: 1 inch  4.5 inches.
B would be correct if there were 100 minutes
Incorrect choices:
in an hour.
A disregards the last half-inch of the carrot.
C is the result of adding an extra hour.
C disregards the last half-inch of the carrot
D is the result of adding an extra hour and
and the fact that it begins at the 1-inch
assuming 100 minutes in an hour.
hash mark.
17. Correct response: A D is incorrect; 5 ​ __12 ​represents the hash mark
(Convert or estimate conversions of measures) at the end of the carrot.
One quart is equal to 4 cups. To find how
many quarts are equal to 10 cups, divide 10 cups 20. Correct response: A
(Estimate and find length, perimeter, and area)
by 4 cups/quart: 10 cups  4 cups/quart 
The perimeter of the triangle is the sum of
​ 12 ​quarts.
2.5 quarts, or 2 __
the lengths of its sides. If Haley uses a total of
96 feet of fencing for the perimeter, the length
Incorrect choices:
of the third side of the triangle is 96 feet minus
B is the number of pints Gary would need the lengths of the other two sides. 96 feet 
(1 pint  2 cups). 24 feet  32 feet  40 feet.
C is the result of subtracting 10 cups 
4 cups. Incorrect choices:
D is the result of multiplying 10 cups  B reflects a mistake in subtraction.
4 cups. C is the sum of the two sides given: 32  24.
D is the result of subtracting only one side
18. Correct response: B (32 ft) from the total: 96  32.
(Select appropriate unit for measuring weight/
mass, capacity, length, perimeter, and area)
21. Correct response: D
A shoe is generally somewhere between 15
(Estimate and find length, perimeter, and area)
and 30 centimeters, so centimeters is a good
The area of the field is the length times the
unit of measurement to use.
width: 100 m  20 m  2,000 m2.

88 Standardized Test Tutor: Math, Grade 4 © Michael Priestley, Scholastic Teaching Resources
21. (continued) 25. Correct response: D
Incorrect choices: (Identify, classify, and describe solid figures and
A is the result of adding 100 m  20 m their attributes)
instead of multiplying. A square pyramid has five faces: the square
base and four triangular sides.
B reflects a mistake in multiplication
(by a power of 10). Incorrect responses:
C is the perimeter of the field instead A A cube has six faces.
of the area.
B A sphere has no faces.
22. Correct response: C C A rectangular prism has six faces.
(Identify and classify acute, obtuse, and right
angles and triangles) 26. Correct response: B
An acute triangle has three angles less (Locate and name points on a coordinate plane
than 90º. using ordered pairs)
In an ordered pair, the first number (x)
Incorrect choices: indicates the number of units across and the
A is a right triangle. second number (y) indicates the number of
units up or down. On the grid, point F is located
B and D are both obtuse triangles. at (4, 6).

23. Correct response: A Incorrect responses:


(Identify, classify, and describe plane figures
A is the result of reading the y coordinate first
and their attributes)
(up 6) and then the x coordinate (4 across).
A rectangle must have two pairs of
parallel sides. C is the result of reading only the y
coordinate (6).
Incorrect choices: D is the result of reading only the x
B The two pairs of sides in a rectangle need coordinate (4).
not be equal in length.
C a nd D are incorrect because rectangles
27. Correct response: A
(Locate and name points on a coordinate plane
have four right angles.
using ordered pairs)
In an ordered pair, the first number (x)
24. Correct response: C
indicates the number of units across and the
(Determine congruence and similarity)
second number (y) indicates the number
The sides of similar figures have the same
of units up or down. On the grid, vertex P is
ratio. Jee’s drawing is twice as tall (2 m) as it
located at (2, 7).
is wide (1 m). The figure in answer choice C is
similar because it is twice as tall (6 m) as it is
Incorrect responses:
wide (3 m).
B Vertex Q is located at (7, 7).
Incorrect choices: C Vertex R is located at (7, 2).
In A, the figure has a ratio of 4 to 1 and is not D Vertex S is located at (2, 2).
similar to Jee’s drawing.
In B, the figure also has a ratio of 4 to 1 and is
not similar to Jee’s drawing.
In D, the figure has a ratio of 5 to 2 and is not
similar to Jee’s drawing.

Standardized Test Tutor: Math, Grade 4 © Michael Priestley, Scholastic Teaching Resources
89
28. Correct response: C 31. Correct response: A
(Identify parallel, intersecting, and (Solve problems involving length, perimeter,
perpendicular lines) area, time, temperature, volume, and
Parallel lines are lines that never intersect. weight/mass)
The lines in answer choice C will never intersect The weight of the stick is the total weight
and thus are parallel. of the stick and rock together (3.3 lb) minus
the weight of the rock (2.4 lb): 3.3 lb  2.4 lb
Incorrect choices:  0.9 lb.
In A
 , the figure contains lines that are perpen-
Incorrect choices:
dicular, not parallel.
B reflects a mistake in subtraction.
In B
 , the figure contains lines that intersect
to form a pair of acute angles and a pair of C is the weight of the rock and stick together.
obtuse angles. D is the result of adding 2.4 lb  3.3 lb instead
In D
 , the figure contains lines that will of subtracting.
eventually intersect.
32. Correct response: C
(Solve problems involving length, perimeter,
29. Correct response: D
area, time, temperature, volume, and
(Identify lines of symmetry)
weight/mass)
The drawing of a candy has one vertical
The perimeter of the field is two times
line of symmetry and one horizontal line of
the length plus two times the width: 2(50 m)
symmetry, giving it two lines of symmetry.
 2(30 m)  160 m.
Incorrect responses:
Incorrect choices:
In A
 , the figure has radial symmetry, and thus
A is the result of adding the length and the
has more than two lines of symmetry.
width: 50 m  30 m.
In B
 , the figure has one vertical line of
B is the result of doubling the length and
symmetry.
adding the width: 2(50 m)  30 m.
In C
 , the figure has one horizontal line
D is the area of the parking lot (50 m  30 m)
of symmetry.
instead of the perimeter.
30. Correct response: B 33. Correct response: C
(Transform figures in the coordinate plane, (Match 3-dimensional objects and their
e.g., translation, rotation, reflection) 2-dimensional representations e.g., nets)
The figure in answer choice B has been The figure in answer choice C can be folded
rotated 90º about the point located at (5, 3). to form a cube like the one shown. This may be
demonstrated by cutting this figure from a piece
Incorrect choices:
of paper and folding it.
In A
 , the picture shows the figure translated
2 units to the right. Incorrect choices:
In C
 , the picture shows a reflection of the In A, this figure cannot be folded to form
figure. a cube (it would have only four faces).
In D
 , the picture shows the figure translated In B, this figure cannot be folded to form
down 2 units. a cube (it would have only four faces).
In D, this figure cannot be folded to form
a cube because the faces have not been
drawn in the correct locations.

90 Standardized Test Tutor: Math, Grade 4 © Michael Priestley, Scholastic Teaching Resources
34. Correct response: A 36. (continued)
(Interpret data presented in line, bar, and circle Incorrect choices:
graphs, pictographs, tables, charts) A lists three possible pairs out of the
Nine classmates take the bus to school (9), six choices.
while six walk and two bike (6  2  8), and 9 is
B lists two groupings, the pancake choices
greater than 8.
and the topping choices.
Incorrect choices: D lists all of the possible pairs of letters with-
B The same number of classmates walk (6) as out regard to whether they are pancakes
drive and bike combined (4  2  6). or toppings.
C More classmates take the bus (9) than
walk (6).
37. Correct response: B
(Find probabilities)
D Fewer students bike (2) than drive (3). The spinner is divided into 8 equal spaces,
and 2 of the spaces are blue. The probability that
35. Correct response: B ​ 28 ​, or __
the arrow will land on a blue space is __ ​ 14 ​.
(Interpret data presented in line, bar, and circle
graphs, pictographs, tables, charts) Incorrect choices:
Fernando made $15 in week 3, which is more
A represents the probability that the arrow
than he earned in any other week.
will land on yellow.
Incorrect choices: C represents the probability that the arrow
will land on red.
A The amount of money Fernando earned
increased and decreased across six weeks. D is “1” over the number of blue spaces.
C The greatest decrease in the amount of
money Fernando earned was between 38. Correct response: A
weeks 3 and 4. (Determine and describe the mean, median,
mode, and range of data)
D Fernando earned the least amount of
The median of the data is the number in
money in week 6, not week 4.
the middle of the data set when the data are
listed from least to greatest. When the weights
36. Correct response: C of Julio’s dogs are listed from least to greatest
(List all possible outcomes or construct sample (23, 38, 48, 54, 82), 48 is in the middle.
spaces using lists, charts, frequency tables, and
tree diagrams) Incorrect choices:
The customer chooses one of the two types
B is incorrect; 49 is the mean of the data
of pancake and one of the four toppings. There
(245  5).
are eight possible outcomes (2  4), because
each of the two types of pancakes can be paired C is incorrect; 59 is the range of the data
with any of the four toppings. (82  23).
D is incorrect; 245 is the sum of the
five weights.

39. Correct response: C


(Determine and describe the mean, median,
mode, and range of data)
The range is the difference between the
highest temperature (32F) and the lowest
temperature (14F): 32F  14F  18F.

Standardized Test Tutor: Math, Grade 4 © Michael Priestley, Scholastic Teaching Resources
91
39. (continued) 42. (continued)
Incorrect choices: Incorrect choices:
A is the difference between the last A, B, and C are incorrect; they are not
temperature listed (27F) and the first equivalent to ($25  $5)  m  $50, and
temperature listed (16F). they all represent misunderstandings of
B is the difference between the highest the basic properties of operations.
temperature and the second lowest:
32F  16F. 43. Correct response: A
(Use algebraic expressions, patterns, and
D is the sum of all the temperatures listed. simple equations and inequalities to represent
problem situations)
40. Correct response: D
Terry started with c crackers and then gave
(Collect, organize, display, and interpret data to
9 to Diana. This situation is represented by the
solve problems)
expression c  9. After he gave 9 crackers to
The circle graph shows that __ ​ 12 ​of the students
Diana, he had 14 left. That means that c  9  14.
chose pencils; ​ __12 ​of the total number of students
(24) is 12. Incorrect choices:
B The equation represents a situation in
Incorrect choices:
which Terry started with 14 crackers, gave
A is the denominator of the fraction of away 9, and c is the number of crackers he
students who chose pencils. had left.
B is the number of students who chose C The equation represents a situation in
markers (​ __16 ​of 24). which Terry started with c crackers, got
C is the number of students who chose 9 more, and ended up with 14.
pens (​ __13 ​of 24). D The equation represents a situation in
which Terry started with 14 crackers,
41. Correct response: B gave away some number (c), and ended
(Identify, describe, and extend numerical and up with 9.
geometric patterns)
In the number pattern, each number is 2 44. Correct response: C
times as large as the number before it. The next (Represent and describe mathematical
number in the pattern will be 16  2. relationships with lists, tables, charts, graphs,
and diagrams)
Incorrect choices:
Answer choice C shows the relationship
A is the result of subtracting the first between the number of minutes and the cost
two numbers in the pattern (4  2) of the call. For each number of minutes, the cost
to determine that the rule is “ 2.” is one-half, or 0.5, that number. For example,
C is the result of subtracting the last the cost of a call that lasts 4 minutes is 0.5  4,
two numbers in the pattern (16  8) or $2.00.
to determine that the rule is “ 8.”
Incorrect choices:
D reflects a misunderstanding of the number
pattern or how to describe it. A shows that the number of minutes and the
cost of the call are the same (the cost would
be $1.00 per minute).
42. Correct response: D
(Apply basic properties and order of operations B shows that the cost for each minute is
with algebraic expressions) $1.50.
By the associative property, ($25  $5)  m D shows that the total cost of a call is $0.50
 $50 is equivalent to $25  ($5  m)  $50. per minute plus $0.50.
92 Standardized Test Tutor: Math, Grade 4 © Michael Priestley, Scholastic Teaching Resources
45. Correct response: B
(Evaluate the reasonableness of a solution)
The total cost of the tickets will be the
number of tickets (62) times the cost of one
ticket ($8.95). By rounding these numbers to
the nearest ten (60) and the nearest whole
number ($9), you can determine that a reason-
able estimate of the cost is 60  $9, or $540.

Incorrect choices:
In A, the estimate is too high by almost
a power of 10.
In C, the estimate is too low and may be
based on 60  $5.
In D, the estimate is too low and may be
based on adding 60  $10 instead of
multiplying.

Standardized Test Tutor: Math, Grade 4 © Michael Priestley, Scholastic Teaching Resources
93
Standardized Test Tutor: Math
Student Scoring Chart
Grade 4
Student Name

Teacher Name

No. Correct/
Test 1 Item Numbers Percent (%)
Total
Number and Number Sense 1–6 /6
Operations 7–15 /9
Measurement and Geometry 16–33 /18
Statistics and Probability 34–40 /7
Patterns, Relations, and Algebra 41–45 /5
Total 1–45 /45

No. Correct/
Test 2 Item Numbers Percent (%)
Total
Number and Number Sense 1–6 /6
Operations 7–15 /9
Measurement and Geometry 16–33 /18
Statistics and Probability 34–40 /7
Patterns, Relations, and Algebra 41–45 /5
Total 1–45 /45

No. Correct/
Test 3 Item Numbers Percent (%)
Total
Number and Number Sense 1–6 /6
Operations 7–15 /9
Measurement and Geometry 16–33 /18
Statistics and Probability 34–40 /7
Patterns, Relations, and Algebra 41–45 /5
Total 1–45 /45

Comments/Notes:

94 Standardized Test Tutor: Math, Grade 4 © Michael Priestley, Scholastic Teaching Resources
Standardized Test Tutor: Math Grade 4
Classroom Scoring Chart
Teacher Name

Student Name Test 1 Test 2 Test 3

Standardized Test Tutor: Math, Grade 4 © Michael Priestley, Scholastic Teaching Resources
95
Notes:

96 Standardized Test Tutor: Math, Grade 4 © Michael Priestley, Scholastic Teaching Resources

You might also like